Sie sind auf Seite 1von 82

Case Digest: Estrada vs Escritor 492 SCRA 1 AM No P-02-1651

Estrada vs. Escritor,


492 SCRA 1, A.M. No. P-02-1651, August 4, 2003

Facts:
Escritor is the Court Interpreter of RTC Branch 253 of Las Pias City. Estrada requested an
investigation of respondent for cohabiting with a man not her husband and having a child with the
latter while she was still married.Estrada believes that Escritor is committing a grossly immoral act
which tarnishes the image of the judiciary, thus she should not be allowed to remain employed
therein as it might appear that the court condones her act.
Escritor admitted the above-mentioned allegations but denies any liability for the alleged gross
immoral conduct for the reason that she is a member of the religious sect Jehovahs Witness and
Watch Tower Society and her conjugal arrangement is approved and is in conformity with her
religious beliefs. She further alleged that they executed a Declaration of Pledging Faithfulness in
accordance with her religion which allows members of Jehovahs Witnesses who have been
abandoned by their spouses to enter into marital relations. The Declaration makes the union moral
and binding within the congregation throughout the world except in countries where divorce is
allowed.

Issue:
Is Escritor guilty of gross immorality for having an illicit relationship?
Does her religious belief justify such act?

Ruling:
Yes the act was grossly immoral. In a catena of cases, the Court has ruled that government
employees engaged in illicit relations are guilty of "disgraceful and immoral conduct" for which he/
she may be held administratively liable. In these cases, there was not one dissent to the majority's
ruling that their conduct was immoral. The respondents themselves did not foist the defense that
their conduct was not immoral, but instead sought to prove that they did not commit the alleged act
or have abated from committing the act.
No, Escritor is not guilty of gross immorality and she cannot be penalized for her freedom of religion
justifies her conjugal arraignment. In interpreting the Free Exercise Clause, the realm of belief poses
no difficulty. The early case of Gerona v. Secretary of Education is instructive on the matter, viz:
The realm of belief and creed is infinite and limitless bounded only by one's imagination and thought.
So is the freedom of belief, including religious belief, limitless and without bounds. One may believe
in most anything, however strange, bizarre and unreasonable the same may appear to others, even
heretical when weighed in the scales of orthodoxy or doctrinal standards. But between the freedom
of belief and the exercise of said belief, there is quite a stretch of road to travel.

The Court recognizes that state interests must be upheld in order that freedom, including religious
freedom, may be enjoyed.
Garcia vs. J. Drilon and Garcia, G. R.
No. 179267, 25 June 2013
posted in RESWRI2 cases by katcobing
Nature of the Case: Petition for Review of Republic Act (R.A.) 9262

Facts: Private respondent Rosalie filed a petition before the RTC of Bacolod City a Temporary Protection
Order against her husband, Jesus, pursuant to R.A. 9262, entitled An Act Defining Violence Against Women and
Their Children, Providing for Protective Measures for Victims, Prescribing Penalties Therefor, and for Other
Purposes. She claimed to be a victim of physical, emotional, psychological and economic violence, being
threatened of deprivation of custody of her children and of financial support and also a victim of marital infidelity on
the part of petitioner.

The TPO was granted but the petitioner failed to faithfully comply with the conditions set forth by the said TPO,
private-respondent filed another application for the issuance of a TPO ex parte. The trial court issued a modified
TPO and extended the same when petitioner failed to comment on why the TPO should not be modified. After the
given time allowance to answer, the petitioner no longer submitted the required comment as it would be an axercise
in futility.

Petitioner filed before the CA a petition for prohibition with prayer for injunction and TRO on, questioning the
constitutionality of the RA 9262 for violating the due process and equal protection clauses, and the validity of the
modified TPO for being an unwanted product of an invalid law.

The CA issued a TRO on the enforcement of the TPO but however, denied the petition for failure to raise the issue
of constitutionality in his pleadings before the trial court and the petition for prohibition to annul protection orders
issued by the trial court constituted collateral attack on said law.

Petitioner filed a motion for reconsideration but was denied. Thus, this petition is filed.

Issues: WON the CA erred in dismissing the petition on the theory that the issue of constitutionality was not raised
at the earliest opportunity and that the petition constitutes a collateral attack on the validity of the law.

WON the CA committed serious error in failing to conclude that RA 9262 is discriminatory, unjust and violative of
the equal protection clause.

WON the CA committed grave mistake in not finding that RA 9262 runs counter to the due process clause of the
Constitution

WON the CA erred in not finding that the law does violence to the policy of the state to protect the family as a basic
social institution

WON the CA seriously erredin declaring RA 9262 as invalid and unconstitutional because it allows an undue
delegation of judicial power to Brgy. Officials.

Decision: 1. Petitioner contends that the RTC has limited authority and jurisdiction, inadequate to tackle the
complex issue of constitutionality. Family Courts have authority and jurisdiction to consider the constitutionality of
a statute. The question of constitutionality must be raised at the earliest possible time so that if not raised in the
pleadings, it may not be raised in the trial and if not raised in the trial court, it may not be considered in appeal.
2. RA 9262 does not violate the guaranty of equal protection of the laws. Equal protection simply requires that all
persons or things similarly situated should be treated alike, both as to rights conferred and responsibilities imposed.
In Victoriano v. Elizalde Rope Workerkers Union, the Court ruled that all that is required of a valid classification is
that it be reasonable, which means that the classification should be based on substantial distinctions which make for
real differences; that it must be germane to the purpose of the law; not limited to existing conditions only; and apply
equally to each member of the class. Therefore, RA9262 is based on a valid classification and did not violate the
equal protection clause by favouring women over men as victims of violence and abuse to whom the Senate extends
its protection.

3. RA 9262 is not violative of the due process clause of the Constitution. The essence of due process is in the
reasonable opportunity to be heard and submit any evidence one may have in support of ones defense. The grant of
the TPO exparte cannot be impugned as violative of the right to due process.

4. The non-referral of a VAWC case to a mediator is justified. Petitioners contention that by not allowing
mediation, the law violated the policy of the State to protect and strengthen the family as a basic autonomous social
institution cannot be sustained. In a memorandum of the Court, it ruled that the court shall not refer the case or any
issue therof to a mediator. This is so because violence is not a subject for compromise.

5. There is no undue delegation of judicial power to Barangay officials. Judicial power includes the duty of the
courts of justice to settle actual controversies involving rights which are legally demandable and enforceable and to
determine whether or not there has been a grave abuse of discretion amounting to lack or excess of jurisdiction on
any part of any branch of the Government while executive power is the power to enforce and administer the laws.
The preliminary investigation conducted by the prosecutor is an executive, not a judicial, function. The same holds
true with the issuance of BPO. Assistance by Brgy. Officials and other law enforcement agencies is consistent with
their duty executive function.

The petition for review on certiorari is denied for lack of merit.

People v Echegaray G.R. No. 117472. February 7, 1997

Per Curiam

Facts:

The SC rendered a decision in the instant case affirming the conviction of the accused-appellant for
the crime of raping his ten-year old daughter. The crime having been committed sometime in April,
1994, during which time Republic Act (R.A.) No. 7659, commonly known as the Death Penalty Law,
was already in effect, accused-appellant was inevitably meted out the supreme penalty of death.
The accused-appellant timely filed a Motion for Reconsideration which focused on the sinister motive
of the victim's grandmother that precipitated the filing of the alleged false accusation of rape against
the accused. This was dismissed.

On August 6, 1996, accused-appellant discharged the defense counsel, Atty. Julian R. Vitug, and
retained the services of the Anti-Death Penalty Task Force of the Free Legal Assistance Group of the
Philippines.

A supplemental Motion for Reconsideration prepared by the FLAG on behalf of accused-appellant.


In sum, the Supplemental Motion for Reconsideration raises three (3) main issues: (1) mixed factual
and legal matters relating to the trial proceedings and findings; (2) alleged incompetence of accused-
appellant's former counsel; and (3) purely legal question of the constitutionality of R.A. No. 7659.

Issue: Whether or not Article III, Section 19 (1) absolutely abolished the death penalty.

Ratio:

One of the indispensable powers of the state is the power to secure society against threatened and
actual evil. Pursuant to this, the legislative arm of government enacts criminal laws that define and
punish illegal acts that may be committed by its own subjects, the executive agencies enforce these
laws, and the judiciary tries and sentences the criminals in accordance with these laws.

The opposition to the death penalty uniformly took the form of a constitutional question of whether or
not the death penalty is a cruel, unjust, excessive or unusual punishment in violation of the
constitutional proscription against cruel and unusualpunishments.

Harden- "The penalty complained of is neither cruel, unjust nor excessive. In Ex-parte Kemmler, 136
U.S., 436, the United States Supreme Court said that 'punishments are cruel when they involve
torture or a lingering death, but the punishment of death is not cruel, within the meaning of that word
as used in the constitution. It implies there something inhuman and barbarous, something more
than the mere extinguishment of life.

Limaco- "x x x there are quite a number of people who honestly believe that the supreme penalty is
either morally wrong or unwise or ineffective. However, as long as that penalty remains in the
statute books, and as long as our criminal law provides for its imposition in certain cases, it is the
duty of judicial officers to respect and apply the law regardless of their private opinions,"

Munoz- A reading of Section 19 (1) of Article III will readily show that there is really nothing therein
which expressly declares the abolition of the death penalty. The provision merely says that the
death penalty shall not be imposed unless for compelling reasons involving heinous crimes the
Congress hereafter provides for it and, if already imposed, shall be reduced to reclusion perpetua.
The language, while rather awkward, is still plain enough

Nothing is more defining of the true content of Article III, Section 19 (1) of the 1987 Constitution
than the form in which the legislature took the initiative in re-imposing the death penalty.

The Senate never doubted its power as vested in it by the constitution, to enact legislation re-
imposing the death penalty for compelling reasons involving heinous crimes. Pursuant to this
constitutional mandate, the Senate proceeded to a two-step process consisting of: first, the
decision, as a matter of policy, to re-impose the death penalty or not; and second, the vote to pass
on the third reading the bill re-imposing the death penalty for compelling reasons involving heinous
crimes.

With seventeen (17) affirmative votes and seven (7) negative votes and no abstention, the Chair
declared that the Senate has voted to re-incorporate death as a penalty in the scale of penalties as
provided in the Revised Penal Code.

The import of this amendment is unmistakable. By this amendment, the death penalty was not
completely abolished by the 1987 Constitution. Rather, it merely suspended the death penalty and
gave Congress the discretion to review it at the propitious time.
We have no doubt, therefore, that insofar as the element of heinousness is concerned, R.A. No.
7659 has correctly identified crimes warranting the mandatory penalty of death. As to the other
crimes in R.A. No. 7659 punished by reclusion perpetua to death, they are admittingly no
less abominable than those mandatorily penalized by death. The proper time to determine their
heinousness in contemplation of law, is when on automatic review, we are called to pass on a death
sentence involving crimes punishable by reclusion perpetua to death under R.A. No. 7659, with the
trial court meting out the death sentence in exercise of judicial discretion. This is not to say,
however, that the aggravating circumstances under the Revised Penal Code need be additionally
alleged as establishing the heinousness of the crime for the trial court to validly impose the death
penalty in the crimes under R.A. No. 7659 which are punished with the flexible penalty of reclusion
perpetua to death.

A studious comparison of the legislative proceedings in the Senate and in the House of
Representatives reveals that, while both Chambers were not wanting of oppositors to the death
penalty, the Lower House seemed less quarrelsome about the form of the death penalty bill as a
special law specifying certain heinous crimes without regard to the provisions of the Revised Penal
Code and more unified in the perception of what crimes are heinous and that the fact of their very
heinousness involves the compulsion and the imperative to suppress, if not completely eradicate,
their occurrence. Be it the foregoing general statement of Representative Sanchez or the following
details of the nature of the heinous crimes enumerated in House Bill No. 62
by Representative Miguel L. Romero of Negros Oriental, there was clearly, among the hundred or so
re-impositionists in the Lower House, no doubt as to their cause.

Article III, Section 19 (1) of the 1987 Constitution plainly vests in Congress the power to re-impose
the death penalty "for compelling reasons involving heinous crimes". This power is not subsumed in
the plenary legislative power of Congress, for it is subject to a clear showing of "compelling reasons
involving heinous crimes."

The constitutional exercise of this limited power to re-impose the death penalty entails (1)
that Congress define or describe what is meant by heinous crimes; (2) that Congress specify
and penalize by death, only crimes that qualify as heinous in accordance with the definition
or description set in the death penalty bill and/or designate crimes punishable by reclusion
perpetua to death in which latter case, death can only be imposed upon the attendance of
circumstances duly proven in court that characterize the crime to be heinous in accordance
with the definition or description set in the death penalty bill; and (3) that Congress, in
enacting this death penalty bill be singularly motivated by "compelling reasons involving
heinous crimes."

It is specifically against the foregoing capital crimes that the test of heinousness must be squarely
applied.

We believe, however, that the elements of heinousness and compulsion are inseparable and are, in
fact, interspersed with each other. Because the subject crimes are either so revolting and debasing
as to violate the most minimum of the human standards of decency or its effects, repercussions,
implications and consequences so destructive, destabilizing, debilitating, or aggravating in the
context of our socio-political and economic agenda as a developing nation, these crimes must be
frustrated, curtailed and altogether eradicated.

Article III, Section 19 (1) of the 1987 Constitution simply states that congress, for compelling reasons
involving heinous crimes, may re-impose the death penalty. Nothing in the said provision imposes a
requirement that for a death penalty bill to be valid, a positive manifestation in the form of a higher
incidence of crime should first be perceived and statistically proven following the suspension of the
death penalty. Neither does the said provision require that the death penalty be resorted to as a last
recourse when all other criminal reforms have failed to abate criminality in society.

It is immaterial and irrelevant that R.A. No. 7659 cites that there has been an "alarming upsurge of
such crimes", for the same was never intended by said law to be the yardstick to determine the
existence of compelling reasons involving heinous crimes. Fittingly, thus, what R.A. No. 7659 states
is that "the Congress, in the interest of justice, public order and rule of law, and the need to
rationalize and harmonize the penal sanctions for heinous crimes, finds compelling reasons to
impose the death penalty for said crimes."

LITO CORPUZ vs. PEOPLE OF THE


PHILIPPINES G.R. No. 180016, April 29,
2014 PERALTA, J.:
JULY 8, 2014 / ARDYESGUERRA

FACTS:
Accused Corpuz received from complainant Tangcoy pieces of jewelry with an
obligation to sell the same and remit the proceeds of the sale or to return the
same if not sold, after the expiration of 30 days.
The period expired without Corpuz remitting anything to Tangcoy.
When Corpuz and Tangcoy met, Corpuz promised that he will pay, but to no
avail.
Tangcoy filed a case for estafa with abuse of confidence against Corpuz.
Corpuz argued as follows:
a. The proof submitted by Tangcoy (receipt) is inadmissible for being a mere
photocopy.

b. The information was defective because the date when the jewelry should be
returned and the date when crime occurred is different from the one testified to by
Tangcoy.

c. Fourth element of estafa or demand is not proved.

d. Sole testimony of Tangcoy is not sufficient for conviction

ISSUES and RULING


Can the court admit as evidence a photocopy of document without violating the best
evidence rule (only original documents, as a general rule, is admissible as
evidence)?
Yes. The established doctrine is that when a party failed to interpose a timely
objection to evidence at the time they were offered in evidence, such objection shall
be considered as waived.

Here, Corpuz never objected to the admissibility of the said evidence at the time it
was identified, marked and testified upon in court by Tangcoy. Corpuz also failed to
raise an objection in his Comment to the prosecutions formal offer of evidence and
even admitted having signed the said receipt.

Is the date of occurrence of time material in estafa cases with abuse of confidence?
No. It is true that the gravamen of the crime of estafa with abuse of confidence under
Article 315, paragraph 1, subparagraph (b) of the RPC is the appropriation or
conversion of money or property received to the prejudice of the owner and that the
time of occurrence is not a material ingredient of the crime. Hence, the exclusion of
the period and the wrong date of the occurrence of the crime, as reflected in the
Information, do not make the latter fatally defective.

Further, the following satisfies the sufficiency of information:

1. The designation of the offense by the statute;

2. The acts or omissions complained of as constituting the offense;

3. The name of the offended party; and

4. The approximate time of the commission of the offense, and the place wherein the
offense was committed.

The 4th element is satisfied. Even though the information indicates that the time of
offense was committed on or about the 5th of July 1991, such is not fatal to the
prosecutions cause considering that Section 11 of the same Rule requires a statement
of the precise time only when the same is a material ingredient of the offense.
What is the form of demand required in estafa with abuse of confidence?
Note first that the elements of estafa with abuse of confidence are as follows:
(a) that money, goods or other personal property is received by the offender in trust, or
on commission, or for administration, or under any other obligation involving the duty
to make delivery of, or to return the same;

(b) that there be misappropriation or conversion of such money or property by the


offender or denial on his part of such receipt;

(c) that such misappropriation or conversion or denial is to the prejudice of another;


and

(d) that there is a demand made by the offended party on the offender.

No specific type of proof is required to show that there was demand. Demand need
not even be formal; it may be verbal. The specific word demand need not even be
used to show that it has indeed been made upon the person charged, since even a mere
query as to the whereabouts of the money [in this case, property], would be
tantamount to a demand.

In Tubb v. People, where the complainant merely verbally inquired about the money
entrusted to the accused, the query was tantamount to a demand.

May a sole witness be considered credible?


Yes. Note first that settled is the rule that in assessing the credibility of witnesses, SC
gives great respect to the evaluation of the trial court for it had the unique opportunity
to observe the demeanor of witnesses and their deportment on the witness stand, an
opportunity denied the appellate courts, which merely rely on the records of the case.

The assessment by the trial court is even conclusive and binding if not tainted with
arbitrariness or oversight of some fact or circumstance of weight and influence,
especially when such finding is affirmed by the CA. Truth is established not by the
number of witnesses, but by the quality of their testimonies, for in determining the
value and credibility of evidence, the witnesses are to be weighed not numbered.
PEOPLE VS. FERRER [48 SCRA 382; NOS.L-32613-14;
27 DEC 1972]
Monday, February 09, 2009 Posted by Coffeeholic Writes
Labels: Case Digests, Political Law

Facts: Hon. Judge Simeon Ferrer is the Tarlac trial court judge that declared
RA1700 or the Anti-Subversive Act of 1957 as a bill of attainder. Thus,
dismissing the information of subversion against the following: 1.)Feliciano Co
for being an officer/leader of the Communist Party of the Philippines
(CPP) aggravated by circumstances of contempt and insult to public officers,
subversion by a band and aid of armed men to afford impunity. 2.) Nilo Tayag
and 5 others, for being members/leaders of the NPA, inciting, instigating people
to unite and overthrow the Philippine Government. Attended by Aggravating
Circumstances of Aid or Armed Men, Craft, and Fraud. The trial court is of
opinion that 1.) The Congress usurped the powers of the judge 2.) Assumed
judicial magistracy by pronouncing the guilt of the CPP without any forms of
safeguard of a judicial trial. 3.) It created a presumption of organizational guilt
by being members of the CPP regardless of voluntariness.

The Anti Subversive Act of 1957 was approved 20June1957. It is an act to


outlaw the CPP and similar associations penalizing membership therein, and for
other purposes. It defined the Communist Party being although a political party
is in fact an organized conspiracy to overthrow the Government, not only by
force and violence but also by deceit, subversion and other illegal means. It
declares that the CPP is a clear and present danger to the security of the
Philippines. Section 4 provided that affiliation with full knowledge of the illegal
acts of the CPP is punishable. Section 5 states that due investigation by a
designated prosecutor by the Secretary of Justice be made prior to filing of
information in court. Section 6 provides for penalty for furnishing false
evidence. Section 7 provides for 2 witnesses in open court for acts penalized by
prision mayor to death. Section 8 allows the renunciation of membership to the
CCP through writing under oath. Section 9 declares the constitutionality of the
statute and its valid exercise under freedom if thought, assembly and
association.

Issues:

(1) Whether or not RA1700 is a bill of attainder/ ex post facto law.

(2) Whether or Not RA1700 violates freedom of expression.

Held: The court holds the VALIDITY Of the Anti-Subversion Act of 1957.

A bill of attainder is solely a legislative act. It punishes without the benefit of


the trial. It is the substitution of judicial determination to a legislative
determination of guilt. In order for a statute be measured as a bill of attainder,
the following requisites must be present: 1.) The statute specifies persons,
groups. 2.) the statute is applied retroactively and reach past conduct. (A bill of
attainder relatively is also an ex post facto law.)

In the case at bar, the statute simply declares the CPP as an organized
conspiracy for the overthrow of the Government for purposes of example of
SECTION 4 of the Act. The Act applies not only to the CPP but also to other
organizations having the same purpose and their successors. The Acts focus is
on the conduct not person.

Membership to this organizations, to be UNLAWFUL, it must be shown


thatmembership was acquired with the intent to further the goals of the
organization by overt acts. This is the element of MEMBERSHIP with
KNOWLEDGE that is punishable. This is the required proof of a members direct
participation. Why is membership punished. Membership renders aid and
encouragement to the organization. Membership makes himself party to its
unlawful acts.

Furthermore, the statute is PROSPECTIVE in nature. Section 4 prohibits acts


committed after approval of the act. The members of the subversive
organizations before the passing of this Act is given an opportunity to escape
liability by renouncing membership in accordance with Section 8. The statute
applies the principle of mutatis mutandis or that the necessary changes having
been made.

The declaration of that the CPP is an organized conspiracy to overthrow the


Philippine Government should not be the basis of guilt. This declaration is only a
basis of Section 4 of the Act. The EXISTENCE OF SUBSTANTIVE EVIL justifies
the limitation to the exercise of Freedom of Expression and Association in this
matter. Before the enactment of the statute and statements in the preamble,
careful investigations by the Congress were done. The court further stresses
that whatever interest in freedom of speech and association is excluded in the
prohibition ofmembership in the CPP are weak considering NATIONAL SECURITY
and PRESERVATION of DEMOCRACY.

The court set basic guidelines to be observed in the prosecution under RA1700.
In addition to proving circumstances/ evidences of subversion, the
following elements must also be established:

1. Subversive Organizations besides the CPP, it must be proven that the


organization purpose is to overthrow the present Government of the Philippines
and establish a domination of a FOREIGN POWER. Membershipis willfully and
knowingly done by overt acts.
2 . I n c a s e o f C P P, t h e c o n t i n u e d p u r s u a n c e o f i t s s u b v e r s i v e
purpose.Membership is willfully and knowingly done by overt acts.

The court did not make any judgment on the crimes of the accused under the
Act. The Supreme Court set aside the resolution of the TRIAL COURT.

THE UNITED STATES v. DIAZ-CONDE

42 Phil. 766 (Outline)


G.R. No. L-18208, February 14, 1922 (Resource)

Plaintiff-appellee:THE UNITED STATES

Defendants-appellants:Vicente Diaz Conde and Apolinaria R. De Conde

What happened:

On December 30, 1915, Bartolome Oliveros and Engracia Lianco accomplished


and delivered to the defendants a contract (named Exhibit B) which stated that the
Oliveros and Lianco had borrowed from the latter a sum of three hundred pesos (Php
300), and by virtue of the terms of said contract, Oliveros and Lianco obligated
themselves to pay to the defendants interest at the rate of five percent (5%) per
month, payable within the first ten days of each and every month, the first payment to
be made on the January 10, 1916.

On May 1, 1916, Act no. 2655 or the Usury Law came into effect. The law stated
that that the legal rate of interest for the loan or forbearance of any money, goods or
credits, [] shall be 12% per annum. Any amount of interest paid or to be paid in
excess of that fixed by law is considered usurious, therefore unlawful.

A complaint was filed in the Court of First Instance of the city of Manila on May 6,
1921, charging the defendants with a violation of the Usury Law (Act No. 2655). Upon
said complaint they were arrested, charged, and pleaded not guilty. On September 1,
1921, the case was finally brought on for trial. At the end of the trial, with consideration
to the evidences cited in court, Hon. M. V. del Rosario, judge, found that the
defendants were guilty of the crime charged in the complaint and sentenced each of
them to pay a fine of P120 and, if they cannot meet their debt obligations, the
defendants would suffer subsidiary imprisonment in accordance with the provisions of
the law. From that sentence each of the defendants made an appeal.

Contention of the State:

The lower court, in the course of its opinion, stated that at the time of the execution and delivery of said contract,
there was no law in force in the Philippine Islands that punishes usury. However, the defendants had collected a
usurious rate of interest after the adoption of the Usury Law in the Philippine Islands (Act No. 2655), Therefore,
they were guilty in the violation of that law and should be punished in accordance with its provisions.

Contention of the Accused:


(a) The contract upon which the alleged usurious interest was collected was executed before Act No. 2655 was
adopted.

(b) The time that the said contract was made (December 30, 1915),there was no usury law in force in the
Philippine Islands.
(c) Act No. 2655 did not become effective until the May 1, 1916, or four months and a half after the contract was
executed.

(d) The said law could have no retroactive effect or operation

(e) The said law impairs the obligation of a contract.

For all of said reasons the judgment imposed by the lower court should be revoked; that the complaint should be
dismissed, and that they should each be discharged from the custody of the law.

Ruling of the Supreme Court:

The Supreme Court en banc promulgated on February 14, 1922 its ruling on the case of The United States vs
Vicente Diaz Conde and Apolinaria R. De Conde (G.R. No. L-18208). The court has decided that the acts
complained of by the defendants did not constitute a crime at the time they were committed. A law imposing a new
penalty, liability or disability, or giving a new right of action, must not be construed as having a retroactive effect. It
is an elementary rule of contract that the laws in force at the time of the contract was made must govern its
interpretation and application. Laws must be construed prospectively and not retrospectively. If a contract is legal
at its commencement, it cannot be rendered illegal by any subsequent legislation. If that were permitted, then the
obligations of a contract might be impaired, which is prohibited by Philippine law.

Ex post factolaws, unless they are favorable to the defendant, are prohibited in this jurisdiction. Every law that
makes an action, done before the passage of the law, and which was innocent when done, criminal, and punishes
such action, is an ex post facto law. The Legislature is prohibited from adopting a law which will make an act done
before its adoption a crime, as in the case of Act No. 2655. A law may be given a retroactive effect in civil action,
providing it is curative in character, but ex post facto laws are absolutely prohibited unless its retroactive effect is
favorable to the defendant.

The complaint was therefore dismissed, and the defendants were discharged from the custody of the law with
costs.

EN BANC [G.R. No. L-1960. November 26, 1948.] THE PEOPLE OF THE PHILIPPINES, plaintiff-
appellee, vs. FLORENTINO ABILONG, defendant-appellant. FACTS: That on or about the 17th day
of September, 1947, in the City of Manila, Philippines, Florentino Abilong, the accused, being then a
convict sentenced and ordered to serve destierro during which he should not enter any place within
the radius of 100 kilometers from the City of Manila for attempted robbery, evaded the service of said
sentence by going beyond the limits made against him and commit vagrancy. ISSUE: Whether the
lower court erred in imposing a penalty on the accused under article 157 of the Revised Penal Code,
which does not cover evasion of service of "destierro." RULING: It is clear that the word
"imprisonment" used in the English text is a wrong or erroneous translation of the phrase "sufriendo
privacion de libertad" used in the Spanish text. It is equally clear that although the Solicitor General
impliedly admits destierro as not constituting imprisonment, it is a deprivation of liberty, though
partial, in the sense that as in the present case, the appellant by his sentence of destierro was
deprived of the liberty to enter the City of Manila. Under the case of People vs. Samonte, as quoted
in the brief of the Solicitor General that "it is clear that a person under sentence of destierro is
suffering deprivation of his liberty and escapes from the restrictions of the penalty when he enters
the prohibited area."
PEOPLE v. FORMIGONES
November 29, 1950 (G.R. No. L-3246)

PARTIES:
plaintiff-appellee: THE PEOPLE OF THE PHILIPPINES
defendant-appellant: ABELARDO FORMIGONES

FACTS:
From November to December 1946, defendant Abelardo Formigones together with his wife Julia Agricola,
and his five children lived in the house of his half-brother, Zacarias Formigones to find employment as
harvesters. One afternoon, the accused, without any previous quarrel or provocation whatsoever, took his
bolo from the wall of the house and stabbed his wife at the back, the blade penetrating the right lung
which latter caused her death. When she fall ont he ground the defendant carried her up the house, laid
her on the floor of the living room and then lay down beside her. He was convicted of parricide and was
sentenced to prison. The defendant entered a plea of not guilty. His counsel presented testimonies of two
guards of the provincial jail where defendant was confined. They said that he behaved like an insane
person, that sometimes he would remove his clothes in front of others, would not take a bath, and
remained silent and indifferent to his surroundings. His counsel claimed that e is an imbecile therefore
exempt from criminal liability. Dr. Francisco Gomez told that Abelardo was suffering only from
feeblemindedness and not imbecility and that he could distinguish right from wrong. An imbecile so as to
be exempt from criminal liability, he must be deprived completely of reason or discernment and freedom
of the will at the time of committing the crime.

ISSUE: WON the defendant who is suffering from feeblemindedness is exempt from criminal liability.

HELD:
No. In order that an exempting circumstance may be taken into account, it is necessary that there be a
complete deprivation of intelligence in committing the act, that is, that the accused be deprived of reason;
that there be no responsibility for his own acts; that he acts without the least discernment; that there be
a complete absence of the power to discern, or that there be a total deprivation of freedom of the will.
As to the strange behaviour of the accused during his confinement, assuming that it was not feigned to
stimulate insanity, it may be attributed either to his being feebleminded or eccentric, or to a morbid
mental condition produced by remorse at having killed his wife. He could distinguish right from wrong.

LADONGA VS PEOPLE
Evangeline Ladonga vs. People of the Philippines
GR no. 141066, February 17, 2005

Facts:

In 1989, spouses Adronico and Evangeline Ladonga, petitioner, became the regular
customer of Alfredo Oculam in his pawnshop. Sometime in May 1990, the Ladonga spouses
obtained a loan from him, guaranteed by check of United Coconut Planters Bank, issued by
Adronico. On last week of April 1990 and during the first week of May 1990 the spouses
obtained additional loan guaranteed by UCPD. And between May and June 1990, the spouses
obtained the third loan guaranteed by UCPD. The three checks bounced upon presentment for
the reason that the account was closed. When the spouses failed to redeem the check,
despite repeated demands, Oculam filed a criminal complaint against them.
While admitting that the checks issued by Adronico bounced because there was no
sufficient deposit or the account was closed, the spouses claim that the checks were issued
only to guarantee the obligation, with an agreement that Oculam should not encash the
checks when they mature, and, that petitioner is not a signatory of the checks and had no
participation in the issuance thereof.

Issue:

Whether or not the petitioner, who was not the issuer of the three checks that
bounced, could be held liable for violation of Batas Pambansa Bilang 22 as conspirator.

Ruling:

Article 8 of the Revised Penal Code provides that a conspiracy exist when two or more
persons come to an agreement concerning the commission of a felony and decide to commit
it. To be held liable guilty as co-principal by reason of conspiracy, the accused must be
shown to have perform an overt act in pursuance or furtherance of the complicity.
It was not proven by direct evidence; petitioner was merely present at the time of the
issuance of the checks. However, this inference cannot be stretched to mean concurrence
with the criminal design. Conspiracy must be established, not by conjectures, but by positive
and conclusive evidence.

People vs. Silvestre and Atienza (Crim1)


People of the Philippine Islands, plaintiff-appellee, vs. Romana Silvestre and Martin Atienza, defendants-
appellants.

En Banc
Villareal, December 14, 1931
Topic: Elements of criminal liability (Art 3.) -- Physical element -- Act/Omission

Facts:
Romana Silvestre is the wife of Domingo Joaquin by his second marriage
Romana cohabited with codefendant Martin Atienza from March 1930 in Masocol, Paombong,
Bulacan
On May 16, 1930, Domingo filed with the justice of the peace for Paombong, Bulacan a sworn
complaint for adultery
After being arrested and released on bail, the two defendants begged the municipal president of
Paombong to speak to the complainant and urge him to withdraw the complaint
The two accused bound themselves to discontinue cohabitation and promised not to live again in
Masocol (Atienza signed the promise)
On May 20, 1930, Domingo Joaquin filed a motion for the dismissal of his complaint and the
justice of the peace dismissed the adultery case
The accused left Masocol and wen to live in Santo Nio, in Paombong
About November 20, 1930: Romana met her son by her former marriage, Nicolas de la Cruz, in
Santo Nio and followed him home to Masocol (under the pretext of asking him for some nipa
leaves)
Martin Atienza, who continued to cohabit with Romana, followed her and lived in the home of
Nicolas
On the night of November 25, 1930, while Nicolas, his wife Antonia, and the appellants were
gathered after supper, Martin told Nicolas and Antonia to take their furniture out of the house
because he was going to set fire to it
He said that that was the only way he could be revenged upon the people of Masocol
who, he said, had instigated the charge of adultery against him and Romana
Martin was armed with a pistol so no one dared say anything to him
Nicolas and Antonia went to ask for help but were too late
The fire destroyed about 48 houses
Witnesses saw Martin and Romana leaving the house on fire
The Court of First Instance of Bulacan convicted Martin and Romana of arson
Martin was convicted as principal by direct participation (14 years, 8 months, and 1 day of
cadena temporal)
Romana was convicted as accomplice (6 years and 1 day of presidio mayor)
The court-appointed counsel for the accused-appellant prays for the affirmance of the CFI
decision with regard to Martin, but assigns errors with reference to Romana:
The lower court erred in convicting Romana as acoomplice
The court erred in not acquitting Romana upon ground of insufficient evidence, or at
least, of reasonable doubt
Issue:
Whether or not Romana can be convicted as accomplice
Holding:
No.
Ratio:
Art. 14 of the Penal Code, in connection with Art. 13 defines an accomplice to be one who does
not take a direct part in the commission of the act, who does not force or induce other to commit
it, nor cooperates in the commission of the act by another act without which it would not have
been accomplished, yet cooperates in the execution of the act by previous or simultaneous
actions.
In the case of Romana: there is no evidence of moral or material cooperation and none of an
agreement to commit the crime in question. Her mere presence and silence while they are
simultaneous acts, do not constitute cooperation, for it does not appear that they encouraged or
nerved Martin Atienza to commit the crime of arson; and as for her failure to give the alarm, that
being a subsequent act it does not make her liable as an accomplice.
Mere passive presence at the scene of another's crime, mere silence and failure to give the
alarm, without evidence of agreement or conspiracy, do not constitute the cooperation required by
Art. 14 of the Penal Code for complicity in the commission of the crime witnessed passively, or
with regard to which one has kept silent
Decision is affirmed with reference to Martin Atienza, reversed with reference to Romana Silvestre, who is
acquitted.
People vs. Talingdan (Crim1)
The People of the Philippines, plaintiff-appellee, vs. Nemesio Talingdan, Magellan Tobias, Augusto
Berras, Pedro Bides and Teresa Domogma, accused-appellants

En Banc
Per Curiam, July 6, 1978

Topic: Elements of criminal liability (Art. 3) -- Physical element -- Act/Omission


Facts:
Teresa Domogma was the supposed wife of the deceased Bernardo Bagabag
No certificate or any other proof of their marriage could be presented by the prosecution
They lived with their children in Sobosob, Salapadan, Abra
Their relationship had been strained and beset with troubles for Teresa had deserted her
family home a couple of times and each time Bernardo took time out to look for her
On 2 different occasions, appellant Nemesis Talingdan has visited Teresa in their house while
Bernardo was out at work, and during those visits Teresa had made Corazon, their then 12-year
old daughter to go down the house and leave them
Bernardo had gotten wind that an illicit relationship was going on between Talingdan and Teresa
About a month before Bernardo was killed, Teresa had again left their house and did not come
back for a period of more than 3 weeks, and Bernardo came to know later that she and Talingdan
were seen together in the town of Tayum Abra during that time
Just two days before Bernardo was killed (Thursday), Bernardo and Theresa had a violent
quarrel; Bernardo slapped Theresa several times, resulting in Theresa seeking the help of the
police
Accused Talingdan, a policeman, came armed to the vicinity of Bernardo's house and called him
to come down; Bernardo ignored him; Talingdan instead left and warned Bernardo that someday
he would kill him
On Saturday, June 24, 1967, Bernardo was gunned down in his house
The defendants' and Corazon's accounts of what happened had variations
Corazon's version:
Friday morning: Corazon was in a creek to wash clothes. She saw her mother Teresa meeting
with Talingdan and their co-appellants Magellan Tobias, Augusto Berras, and Pedro Bides in a
small hut owned by Bernardo
She heard one of them say "Could he elude a bullet"
When Teresa noticed Corazon, she shoved her away saying "You tell your father that we will kill
him"
Saturday, after sunset: Corazon was cooking food for supper when she saw her mother go down
the house to go to the yard where she again met with the other appellants.
She noted the long guns the appellants were carrying.
Teresa came back to the house and proceeded to her room.
Corazon informed Bernardo, who was then working on a plow, about the presence of persons
downstairs, but Bernardo paid no attention
Bernardo proceeded to the kitchen and sat himself on the floor near the door
He was suddenly fired upon form below the stairs of the batalan
The four accused climbed the stairs of the batalan and upon seeing that Bernardo was still alive,
Talingdan and Tobias fired at him again
Bides and Berras did not fire at that precise time but when Corazon tried to call for helo,
Bides warned her that he will kill her if she calls for help
Teresa came out of her room and when Corazon informed her that she recognized the killers, the
former threatened to kill the latter if she reveals the matter to anyone
The defendants'' version:
Teresa loved Bernardo dearly, they never quarreled, and her husband never maltreated her.
Teresa came to know Talingdan only when the latter became a policeman in Sallapadan; an illicit
relationship never existed between them
Talingdan was not in Sallapadan at the time of the killing on June 24; he escorted the Mayor in
Bangued from June 22 to June 26
Tobias, Bides, and Berras claimed to be in the house of one Mrs. Bayongan in Sallapadan,
250-300 meters from the place of the killing
Issue:
Whether or not Teresa Domogma is an accessory to Bernardo's murder
It is contended that there is no evidence proving that she actually joined in the
conspuracy to kill her husband because there is no showing of actual cooperation on her
part with co-appellants in their culpable acts that led to his death
It is claimed that what is apparent is "mere cognizance, acquiescence or approval thereof
on her part, which it is argued is less than what is required for her conviction as a
conspirator
Holding:
Yes. She is an accessory to Bernardo's murder.
Ratio:
Note: The court believed Corazon's testimony.
It is true that proof of her direct participation in the conspiracy is not beyond reasonable doubt;
she cannot have the same liability as her co-appellants. She had no hand in the actual shooting.
It is also not clear if she helped directly in the planning and preparation thereof. But the court is
convinced that she knew it was going to be done and did not object.
There is in the record morally convincing proof that she is at the very least an accessory to the
offense committed.
She did not only order her daughter not to reveal what she knew to anyone, she also claimed to
have no suspects in mind when the peace officers came into their house later to investigate
Whereas before the actual shooting she was more or less passive in her attitude regarding the
conspiracy, after Bernardo was killed, she became active in her cooperation with her co-
appellants
These acts constitute "concealing or assisting in the escape of the principal in the crime"

Male appellants sentenced to death. Guilty beyond reasonable doubt is Teresa Domogma, sentenced to
suffer the indeterminate penalty of 5 years of prision correccional as minimum to 8 years of prision mayor
as maximum.

Manuel v. People
G.R. No. 165842 November 29, 2005

Lesson: Felony, Bigamy, Judicial Declaration of Presumptive Death, malice, good faith as a valid
defense
Actus non facit reum, nisi mens sit rea

Laws Applicable: Art. 3 par 2 RPC, Art. 349 RPC, Art. 41 FC

FACTS:
July 28, 1975: Eduardo married Rubylus Gaa before Msgr. Feliciano Santos in Makati
o Rubylus was charged with estafa in 1975 and thereafter imprisoned
o Eduardo only visited 3 times and never saw her again
January 1996: Eduardo met Tina B. Gandalera, 21 year old computer secretarial student, in
Dagupan City while she looked for a friend during her 2 days stay
Later, Eduardo visited Tina, they went to a motel together and he proposed marriage and
introduced her to his parents who assures that he is single
April 22, 1996: Eduardo married Tina before Judge Antonio C. Reyes, the Presiding Judge of the
RTC of Baguio City and they were able to build a home after
1999: Eduardo only visited their home twice or thrice a year and whenever jobless Tina would
ask for money, he would slap her
January 2001: Eduardo packed his things and left and stopped giving financial support
August 2001: Tina through inquiries from the National Statistics Office (NSO) in Manila and was
embarrassed and humiliated to learn that Eduardo was previously married
Eduardo claimed that he did NOT know that he had to go to court to seek for the nullification of
his first marriage before marrying Tina
RTC: Eduardo guilty beyond reasonable doubt of bigamy and sentenced to an indeterminate
penalty of from 6 years and 10 months, as minimum, to 10 years, as maximum and P200,000.00 by
way of moral damages, plus costs of suit
o Eduardos belief, that his first marriage had been dissolved because of his first wifes 20-year
absence, even if true, did not exculpate him from liability for bigamy
Eduardo appealed to the CA contending that he did so in good faith and without any malicious
intent whereas under Article 3 of the Revised Penal Code, there must be malice for one to be
criminally liable for a felony
CA: affirming the decision of the RTC stating that Article 41 of the Family Code should apply that
there should have been a judicial declaration of Gaas presumptive death as the absent spouse and
modified minimum to 2 years and four months

ISSUE: W/N Eduardo is guilty of Bigamy, a felony by dolo (deceit).

HELD: YES. petition is DENIED. CA affirmed

Art. 349. Bigamy. The penalty of prision mayor shall be imposed upon any person who shall
contract a second or subsequent marriage before the former marriage has been legally dissolved, or
before the absent spouse has been declared presumptively dead by means of a judgment rendered
in the proper proceedings.
o The reason why bigamy is considered a felony is to preserve and ensure the juridical tie of
marriage established by law.
o Article 349 of the Revised Penal Code has made the dissolution of marriage dependent not only
upon the personal belief of parties, but upon certain objective facts easily capable of accurate
judicial cognizance, namely, a judgment of the presumptive death of the absent spouse
For the accused to be held guilty of bigamy, the prosecution is burdened to prove the felony:
o (a) he/she has been legally married; and
o (b) he/she contracts a subsequent marriage without the former marriage having been lawfully
dissolved.
The felony is consummated on the celebration of the second marriage or subsequent marriage
Article 3, paragraph 2 of the Revised Penal Code provides that there is deceit when the act is
performed with deliberate intent
o Malice -a mental state or condition prompting the doing of an overt act WITHOUT legal excuse or
justification from which another suffers injury
o When the act or omission defined by law as a felony is proved to have been done or committed by
the accused, the law presumes it to have been intentional
o For one to be criminally liable for a felony by dolo, there must be a confluence of both an evil act
and an evil intent.
Actus non facit reum, nisi mens sit rea
GR: mistake of fact or good faith of the accused is a valid defense in a prosecution for a felony
by dolo; such defense negates malice or criminal intent.
EX: ignorance of the law is not an excuse because everyone is presumed to know the law.
o Ignorantia legis neminem excusat
burden of the petitioner to prove his defense that when he married he was of the well-grounded
belief that his first wife was already dead, as he had not heard from her for more than 20 years since
1975
o failed to discharge his burden since no judicial declaration as proof
Article 41 of the Family Code amended the rules on presumptive death on Articles 390 and 391
of the Civil Code which states that before the spouse present may contract a subsequent marriage,
he or she must institute summary proceedings for the declaration of the presumptive death of the
absentee spouse, without prejudice to the effect of the reappearance of the absentee spouse.
moral damages may be awarded under Article 2219 in relation to Articles 19, 20 and 21 of the
Civil Code for being against public policy as they undermine and subvert the family as a social
institution, good morals and the interest and general welfare of society

People vs. Puno (Crim1)


People of the Philippines, plaintiff-appellee, vs. Isabelo Puno y Guevarra, alias "Beloy," and Enrique
Amurao y Puno, alias "Enry," accused-appellants

En Banc
Regalado, February 17, 1993
Topic: Mental Element (Mens rea) -- Deliberate intent (Dolo) -- General and specific intent
Facts:

January 13, 1988 in QC, at around 5:00 pm: the accused Isabelo Puno, who is the personal
driver of Mrs. Sarmiento's husband (who was then away in Davao purportedly on account of local
election there) arrived at Mrs. Sarmiento's bakeshop in Araneta Ave, QC
He told Mrs. Sarmiento that her own driver Fred had to go to Pampanga on an emergency so
Isabelo will temporarily take his place
When it was time for Mrs. Sarmiento to go home to Valle Verde in Pasig, she got into her
husband's Mercedes Benz with Isabelo driving
After the car turned right on a corner of Araneta Ave, it stopped and a young man, accused
Enrique Amurao, boarded the car beside the driver
Enrique pointed a gun at Mrs. Sarmiento as Isabelo told her that he needs to "get money" from
her
Mrs. Sarmiento had P7,000 on her bag which she handed to the accused
But the accused said that they wanted P100,000 more
The car sped off north towards the North superhighway where Isabelo asked Mrs. Sarmiento to
issue a check for P100,000
Mrs. Sarmiento drafted 3 checks: two P30,000 checks and one P40,000 check
Isabelo then turned the car around towards Metro Manila; later, he changed his mind and turned
the car again towards Pampanga
According to her, Mrs. Sarmiento jumped out of the car then, crossed to the other side of the
superhighway and was able to flag down a fish vendor's van, her dress had blood because
according to her, she fell down on the ground and was injured when she jumped out of the car
The defense does not dispute the above narrative of the complainant except that according to
Isabelo, he stopped the car at North Diversion and freely allowed Mrs. Sarmiento to step out of
the car
He said he even slowed the car down as he drove away, until he saw that his employer
had gotten a ride
He claimed that she fell down when she stubbed her toe while running across the
highway
Issue:
1. Whether or not the accused can be convicted of kidnapping for ransom as charged
2. Whether or not the said robbery can be classified as "highway robbery" under PD No.
532 (Anti-Piracy and Anti-Highway Robbery Law of 1974)
Holding:
1. No.
2. No.
Ratio:
1. There is no showing whatsoever that appellants had any motive, nurtured prior to or
at the time they committed the wrongful acts against complainant, other than the extortion of
money from her under the compulsion of threats or intimidation.
For this crime to exist, there must be indubitable proof that the actual intent of the
malefactors was to deprive the offended party of her liberty
In the case, the restraint of her freedom of action was merely an incident in the
commission of another offense primarily intended by the offenders
This does not constitute kidnapping or serious illegal detention
2. Jurisprudence reveals that during the early part of the American occupation of our
country, roving bands were organized for robbery and pillage and since the then existing law
against robbery was inadequate to cope with such moving bands of outlaws, the Brigandage Law
was passed (this is the origin of the law on highway robbery)
PD No. 532 punishes as highway robbery only acts of robbery perpetrated by outlaws
indiscriminately against any person or persons on Philippine highways and not acts of robbery
committed against only a predetermined or particular victim
The mere fact that the robbery was committed inside a car which was casually
operating on a highway does not make PD No 532 applicable to the case
This is not justified by the accused's intention
Accused-appellants convicted of robbery (indeterminate sentence of 4 years and 2 months or prision
correccional, as minimum, to 10 years of prision mayor. Accused to pay Mrs. Sarmiento P7,000 as actual
damages and P20,000 as moral damages.)

Criminal Law- People vs. Delim


This case is with regard to Art 8 and 13 of the Revised Penal Code

"the act of one is the act of all"

Case of People of the R.P. vs. Delim

G.R. No. 142773 28January2003

FACTS OF THE CASE:


It is due to the automatic review of the decision of the RTC Branch 46
(Urdaneta City) finding the appellants, guilty beyond reasonable doubt
and sentencing them to death for the murder of Modesto Bantas.

Appellants pleaded not guilty to the charge. The appellants and victim
are related for modesto is an adopted son of their father. On
January 23,1999 Marlon, Robert and Ronald Delim charged into the house
and poked a gun at modesto and herded him outside the house. Leon and
Manuel Delim both armed stayed put and made sure that randy and rita
stayed put

Modesto's lifeless body was then found on January 25, 1999. Marlon,
Ronald, and Leon used denial and alibi as their evidence against the
charge.

*alibis are the weakest of all defenses since it is easy to contrive


and difficult to disprove

ISSUES OF THE CASE:

Is conspiracy and treachery present in this case to ensure that murder


can be the crime?

Yes there is:

CONSPIRACY- is determined when two or more persons agree to commit a


felony and decide to commit it. Conspiracy must be proven with the
same quantum of evidence as the felony itself, more specifically by
proof beyond reasonable doubt. It is not essential that there be proof
as to the existence of a previous agreement to commit a crime. It is
sufficient if, at the time of commission of the crime, the accused had
the same purpose and were united in its executed.

appellants acted in unison when they abducted Modesto. So their acts


were synchronized and executed with precision evincing a preconceived
plan to kill Modesto
There is no:

TREACHERY- there is treachery when the offender commits any of the


crimes against person, employing means, methods, or forms in the
execution thereof which tend directly and especially to insure its
execution, without risk to himself arising from the defense which the
offended party might make.(defenceless?)

For it to be appreciated prosecution needs to prove:

a. employment of means of execution which gives the person no


opportunity to defend himself

b. the means of execution is deliberately and consciously adopted

in the appellants case there are no evidence to the particulars on how


Modesto was assaulted and killed and this in fact does mean that
treachery cannot be proven since it cannot be presumed that modesto
was defenseless during the time that he was being attacked and shot at
by the appellants.

Sheer numbers by the appellants when they attacked modesto does not
constitute proof that the three took advantage of their numerical
superiority and their handguns when Modesto was shot and stabbed.

HELD:

APPELLANTS ARE GUILTY BEYOND REASONABLE DOUBT OF THE FELONY OF


HOMICIDE (THE DECISION OF THE LOWER COURTS WERE MODIFIED TO LOWER THE
CRIME FROM MURDER TO HOMICIDE)

Ah Chong 15 Phil. 488


G.R. No. L-5272 March 19, 1910
CARSON, J.

Lesson: mistake of fact, definition of felony

Laws: Article 1 RPC, Art 3 RPC

FACTS:
August 14, 1908 About 10 pm: Ah Chong, a cook was suddenly awakened by some trying to
force open the door of the room. He sat up in bed and called out twice, "Who is there?" He heard no
answer and was convinced by the noise at the door that it was being pushed open by someone bent
upon forcing his way into the room. The defendant, fearing that the intruder was a robber or a thief,
leaped to his feet and called out. "If you enter the room, I will kill you." At that moment he was struck
just above the knee by the edge of the chair (thought to be an unlawful aggression) which had been
placed against the door. Seizing a common kitchen knife which he kept under his pillow, the
defendant struck out wildly at the intruder who, it afterwards turned out, was his roommate, Pascual
who is a house boy or muchacho who in the spirit of mischief was playing a trick on him
Seeing that Pascual was wounded, he called to his employers and ran back to his room to secure
bandages to bind up Pascual's wounds.
There had been several robberies not long prior to the date of the incident, one of which took
place in a house where he was employed as cook so he kept a knife under his pillow for his personal
protection.
trial court held it as simple homicide

ISSUE: W/N defendant can be held criminally responsible who, by reason of a mistake as to the
facts, does an act for which he would be exempt from criminal liability if the facts were as he
supposed them to be, but which would constitute the crime of homicide or assassination if the actor
had known the true state of the facts at the time when he committed the act.

HELD: trial court should be reversed, and the defendant acquitted of the crime
NO.
GR: acts constituting the crime or offense must be committed with malice or with criminal intent in
order that the actor may be held criminally liable
EX: it appears that he is exempted from liability under one or other of the express provisions of
article 8 of the code
Article 1 RPC of the Penal Code is as follows:
Crimes or misdemeanors are voluntary acts and ommissions punished by law.
o A person voluntarily committing a crime or misdemeanor shall incur criminal liability, even though
the wrongful act committed be different from that which he had intended to commit.
o voluntary act is a free, intelligent, and intentional act
o "malice" signifying the intent
o Actus non facit reum nisi mens sit rea - "the act itself does not make man guilty unless his
intention were so
o Actus me incito factus non est meus actus - an act done by me against my will is not my act
GR: courts have recognized the power of the legislature to forbid, in a limited class of cases, the
doing of certain acts, and to make their commission criminal WITHOUT regard to the intent of the
doer
EX: intention of the lawmaker to make the commission of certain acts criminal without regard to
the intent of the doer is clear and beyond question the statute will not be so construed
ignorantia facti excusat applies only when the mistake is committed without fault or carelessness
defendant at the time, he acted in good faith, without malice, or criminal intent, in the belief that
he was doing no more than exercising his legitimate right of self-defense; that had the facts been as
he believed them to be he would have been wholly exempt from criminal liability on account of his
act; and that he can not be said to have been guilty of negligence or recklessness or even
carelessness in falling into his mistake as to the facts, or in the means adopted by him to defend
himself from the imminent danger which he believe threatened his person and his property and the
property under his charge.

Oanis, 74 Phil. 257


G.R. No.L-47722 July 27, 1943
MORAN, J.

Lesson applicable: mitigating circumstances

FACTS:
Captain Godofredo Monsod, Constabulary Provincial Inspector at Cabanatuan, Nueva Ecija, received from
Major Guido a telegram of the following tenor: "Information received escaped convict Anselmo Balagtas
with bailarina and Irene in Cabanatuan get him dead or alive." Captain Monsod accordingly called for his first
sergeant and asked that he be given four men.
The same instruction was given to the chief of police Oanis who was likewise called by the Provincial
Inspector.
Defendants Oanis and Galanta then went to the room of Irene, and an seeing a man sleeping with his back
towards the door where they were, simultaneously or successively fired at him with their .32 and .45 caliber
revolvers. Awakened by the gunshots, Irene saw her paramour already wounded, and looking at the door where
the shots came, she saw the defendants still firing at him. Shocked by the entire scene. Irene fainted; it turned
out later that the person shot and killed was not the notorious criminal Anselmo Balagtas but a peaceful and
innocent citizen named Serapio Tecson, Irene's paramour.
According to Appellant Galanta, when he and chief of police Oanis arrived at the house, the latter asked
Brigida where Irene's room was. Brigida indicated the place, and upon further inquiry as to the whereabouts of
Anselmo Balagtas, she said that he too was sleeping in the same room.
ISSUE: W/N they may, upon such fact, be held responsible for the death thus caused to Tecson

HELD: appellants are hereby declared guilty of murder with the mitigating circumstance
YES.
ignorantia facti excusat, but this applies only when the mistake is committed without fault or carelessness
appellants found no circumstances whatsoever which would press them to immediate action. The person in
the room being then asleep, appellants had ample time and opportunity to ascertain his identity without hazard
to themselves, and could even effect a bloodless arrest if any reasonable effort to that end had been made, as
the victim was unarmed.
"No unnecessary or unreasonable force shall be used in making an arrest, and the person arrested shall not
be subject to any greater restraint than is necessary for his detention."
a peace officer cannot claim exemption from criminal liability if he uses unnecessary force or violence in
making an arrest
The crime committed by appellants is not merely criminal negligence, the killing being intentional and not
accidental. In criminal negligence, the injury caused to another should be unintentional, it being simply the
incident of another act performed without malice.
2 requisites in order that the circumstance may be taken as a justifying one:
1. offender acted in the performance of a duty or in the lawful exercise of a right-present
2. injury or offense committed be the necessary consequence of the due performance of such duty or the
lawful exercise of such right or office.-not present
According to article 69 of the Revised Penal Code, the penalty lower by 1 or 2 degrees than that prescribed
by law shall, in such case, be imposed.

Padilla vs. Dizon (Crim1)


Alexander Padilla, complainant, vs. The Hon. Baltazar R. Dizon, Presiding Judge of the Regional Trial
Court of Pasay City, Branch 113, respondent.
February 23, 1988

Per Curiam

Facts:
Respondent Baltazar R. Dizon acquitted, in his decision, the tourist and accused, Lo Chi Fai,
saying that Lo Chi Fai had no willful intention to violate the law. He also directed the release to Lo
Chi Fai of at least the amount of US$3,000.00 under Central Bank Circular No. 960.
Lo Chi Fai was caught by Customs guard at the Manila International Airport while
attempting to smuggle foreign currency and foreign exchange instruments out of the
country.
An information was filed against Lo Chi Fai with the RTC for violation of Sec. 6, Central
Bank Circular No. 960 with a penal sanction provided by Sec. 1, PD NO. 1883.
Sec. 6, Central Bank Circular No. 960 provides that no person shall take out or
transmit or attempt to take out or transmit foreign exchange in any form out of the
Philippines without an authorization by the Central Bank. Tourists and non-
resident visitors may take out or send out from the Philippine foreign exchange in
amounts not exceeding such amounts of foreign exchange brought in by them.
Tourists and non-resident temporary visitors bringing with them more than
US$3,000.00 or its equivalent in other foreign currencies shall declare their
foreign exchange in the form prescribed by the Central Bank at points of entries
upon arrival in the Philippines.
Sec. 1, P.D. No. 1883 provides that any person who shall engage in the trading
or purchase and sale of foreign currency in violation of existing laws or rules and
regulations of the Central Bank shall be guilty of the crime of black marketing of
foreign exchange and shall suffer the penalty of reclusion temporal (minimum of
12 years and 1 day and maximum of 20 years) and a fine of no less than
P50,000.00.
At the trial, Lo Chi Fai tried to establish that he was a businessman from Hongkong, that
he had come to the Philippines 9 to 10 times to invest in business in the country with his
business associates, and that he and his business associates declared all the money
they brought in and all declarations were handed to and kept by him.
Because of the revolution taking place in Manila during that time, Lo Chi Fai was urged
by his business associates to come to Manila to bring the money out of the Philippines.
Commissioner of Customs, Alexander Padilla, then filed a complaint against Baltazar R. Dizon for
acquitting Lo Chi Fai.
Issue:
Whether or not respondent Baltazar R. Dizon is guilty of gross incompetence or gross ignorance
of the law in holding that the accused, Lo Chi Fai, for violation of Central Bank Circular No. 960,
the prosecution must establish that the accused had the criminal intent to violate the law.
Held:
Yes.
Ratio:
Baltazar R. Dizon ignored the fact that the foreign currency and foreign currency instruments
found in the possession of Lo Chi Fai when he was apprehended at the airport and the amounts
of such foreign exchange did not correspond to the foreign currency declarations presented by Lo
Chi Fai at the trial, and that these currency declarations were declarations belonging to other
people.
In invoking the provisions of the Central Bank Circular No. 960 to justify the release of
US$3,000.00 to Lo Chi Fai, Baltazar R. Dizon again diplayed gross incompetence and gross
ignorance of law. There is nothing in the Central Bank Circular which could be taken as authority
for the trial court to release the said amount of US Currency to Lo Chi Fai.

Magno vs. CA (Crim1)


Oriel Magno, petitioner, vs. Honorable Court of Appeals and People of the Philippines,
respondents.
June 26, 1992
Paras, J:
Facts:
Oriel Magno, lacking fund in acquiring complete set of equipment to make his car
repair shop operational, approached Corazon Teng, Vice President of Mancor
Industries.
VP Teng referred Magno to LS Finance and Management Corporation, advising its Vice
President, Joey Gomez, that Mancor was willing to supply the pieces of equipment
needed if LS Finance could accommodate Magno and and provide him credit facilities.
The arrangement went on requiring Magno to pay 30% of the total amount of the
equipment as warranty deposit but Magno couldn't afford to pay so he requested VP
Gomez to look for third party who could lend him that amount.
Without Magno's knowledge, Corazon was the one who provided that amount.
As payment to the equipment, Magno issued six checks, two of them were cleared and
the rest had no sufficient fund.
Because of the unsuccessful venture, Magno failed to pay LS Finance which then pulled
out the equipment.
Magno was charged of violation of BP Blg. 2 (The Bouncing Checks Law) and found
guilty.

Issue:
Whether or not Magno should be punished for the issuance of the checks in question.
Held:
No
Ratio:
To charge Magno for the refund of a warranty deposit which he did not withdraw as it
was not his own account, it having remained with LS Finance, is to even make him pay
an unjust debt since he did not receive the amount in question. All the while, said
amount was in the safekeeping of the financing company which is managed by the
officials and employees of LS Finance.
Arsenia Garcia vs Court of
Appeals
484 SCRA 617 Criminal Law Felonies Mala In Se vs Mala Prohibita Crimes Defense of Good
Faith

In 1995, Aquilino Pimentel, Jr., ran for the senatorial elections.

Meanwhile, in Alaminos, Pangasinan, Arsenia Garcia was one of the designated election
officers. Garcia was accused by Pimentel of violating the Electoral Reforms Law of 1987.
Pimentel alleged that Garcia decreased Pimentels vote by 5,000 votes.

The trial court found Garcia guilty. On appeal, Garcia invoked that the trial court erred in
ruling that her defense of good faith was not properly appreciated. She averred that due to
the workload given to her during said elections, she got fatigued and that caused the error in
the tabulation of Pimentels votes.

Pimentel argued that the Electoral Reforms Law is a special law hence it is a malum
prohibitum law and therefore, good faith is not a defense.

ISSUE: Whether or not the alleged violation of Garcia of the Electoral Reforms Law is a
malum prohibitum.

HELD: No. Generally, mala in se crimes refer to those felonies in violation of the Revised Penal
Code. However, it must be noted that mala in se are crimes which are inherently immoral.
Hence, even if the crime is punished by a special law, if it is inherently immoral, then it is
still a crime mala in se.

In this case, the said violation of the Electoral Reforms Law is a mala in se crime because it is
inherently immoral to decrease the vote of a candidate. Note also that what is being punished
is the intentional decreasing of a candidates votes and not those arising from errors and
mistakes. Since a violation of this special law is a malum in se, good faith can be raised as a
defense.
However, Garcias defense of good faith was not proven. Facts show that the decreasing of
Pimentels vote was not due to error or mistake. It was shown that she willingly handled
certain duties which were not supposed to be hers to perform. Thats a clear sign that she
facilitated the erroneous entry.

Criminal Law- People of the R.P. vs. Pugay


THIS CASE IS WITH REGARD TO ART. 3(2) & 8(2) OF THE R.P.C.

"A Conspiracy exists when two or more people come to an agreement concerning the
commission of a felony and decide to commit it."

"A man must use common sense, and exercise due reflection in all his acts; it is his duty to be
cautious, careful and prudent, if not from instinct, then through fear of incurring
punishment."

Case of People of the R.P. vs. Pugay


No. L-74324 17November1988

FACTS OF THE CASE:


The accused are pronounced by the RTC of Cavite guilty beyond reasonable doubt for the
crime of murder of Bayani Miranda and sentencing them to a prison term ranging from 12
years (prison mayor) as mimimum to 20 years (prison temporal) as maximum and for samson
to be sentenced to reclusion perpetua.

Miranda and the accused Pugay are friends. Miranda used to run errands for Pugay and they
used to sleep together. On the evening of May 19, 1982 a town fiesta was held in the public
plaza of Rosario Cavite. Sometime after midnight accused Pugay and Samson with several
companions arrived (they were drunk), and they started making fun of Bayani Miranda. Pugay
after making fun of the Bayani, took a can of gasoline and poured its contents on the latter,
Gabion (principal witness) told Pugay not to do the deed. Then Samson set Miranda on fire
making a human torch out of him. They were arrested the same night and barely a few hours
after the incident gave their written statements.

ISSUES OF THE CASE:

Is conspiracy present in this case to ensure that murder can be the crime? If not what are the
criminal responsibilities of the accused?

There isno:
CONSPIRACY- is determined when two or more persons agree to commit a felony and decide
to commit it. Conspiracy must be proven with the same quantum of evidence as the felony
itself, more specifically by proof beyond reasonable doubt. It is not essential that there be
proof as to the existence of a previous agreement to commit a crime. It is sufficient if, at the
time of commission of the crime, the accused had the same purpose and were united in its
executed.
Since there was no animosity between miranda and the accused, and add to the that that the
meeting at the scene of the incident was purely coincidental, and the main intent of the
accused is to make fun of miranda.
Since there is no conspiracy that was proven, the respective criminal responsibility of Pugay
and Samson arising from different acts directed against miranda is individual NOT collective
and each of them is liable only for the act that was committed by him.

**Conspiracy may be implied from concerted action of the assailants in confronting the
victim.

Criminal Responsibilities:
PUGAY: Having failed to exercise diligence necessary to avoid every undesirable consequence
arising from any act committed by his companions who at the same time were making fun of
the deceased. - GUILTY OF RECKLESS IMPRUDENCE RESULTING TO HOMICIDE

SAMSON:Since there areNOsufficient evidence that appears in the record establishing


qualifying circumstances (treachery, conspiracy). And granted the mitigating circumstance
that he never INTENDED to commit so grave a wrong. - GUILTY OF HOMICIDE

HELD:
JUDGEMENT OF THE LOWER COURT WAS AFFIRMED WITH MODIFICATIONS. JUDGEMENT FOR
GUILTY BEYOND REASONABLE DOUBT FOR MURDER WAS LOWERED TO THE ABOVE JUDGEMENTS.

Ivler vs. San Pedro G.R. No. 172716 November 17, 2010
Bill of Rights
Ivler vs. San Pedro

G.R. No. 172716November 17, 2010


FACTS:

Following a vehicular collision in August 2004, petitioner Jason Ivler (petitioner) was charged before the
Metropolitan Trial Court of Pasig City (MTC), with two separate offenses: (1) Reckless Imprudence Resulting in
Slight Physical Injuries for injuries sustained by respondent Evangeline L. Ponce (respondent Ponce); and (2)
Reckless Imprudence Resulting in Homicide and Damage to Property for the death of respondent Ponces husband
Nestor C. Ponce and damage to the spouses Ponces vehicle.

Petitioner posted bail for his temporary release in both cases. On 2004, petitioner pleaded guilty to the
charge on the first delict and was meted out the penalty of public censure. Invoking this conviction, petitioner
moved to quash the Information for the second delict for placing him in jeopardy of second punishment for the same
offense of reckless imprudence.

The MTC refused quashal, finding no identity of offenses in the two cases.

The petitioner elevated the matter to the Regional Trial Court of Pasig City (RTC), in a petition for
certiorari while Ivler sought from the MTC the suspension of proceedings in criminal case, including the
arraignment his arraignment as a prejudicial question.
Without acting on petitioners motion, the MTC proceeded with the arraignment and, because of
petitioners absence, cancelled his bail and ordered his arrest.

Seven days later, the MTC issued a resolution denying petitioners motion to suspend proceedings and
postponing his arraignment until after his arrest. Petitioner sought reconsideration but as of the filing of this petition,
the motion remained unresolved.

ISSUES:

1. Whether petitioner forfeited his standing to seek relief from his petition for certiorari when the MTC
ordered his arrest following his non-appearance at the arraignment in Reckless Imprudence Resulting in Slight
Physical Injuries for injuries sustained by respondent; and

2. Whether petitioners constitutional right under the Double Jeopardy Clause bars further proceedings in
Reckless Imprudence Resulting in Homicide and Damage to Property for the death of respondent Ponces husband.

RULING:

The accused negative constitutional right not to be "twice put in jeopardy of punishment for the same
offense" protects him from, among others, post-conviction prosecution for the same offense, with the prior verdict
rendered by a court of competent jurisdiction upon a valid information.

Petitioner adopts the affirmative view, submitting that the two cases concern the same offense of reckless
imprudence. The MTC ruled otherwise, finding that Reckless Imprudence Resulting in Slight Physical Injuries is an
entirely separate offense from Reckless Imprudence Resulting in Homicide and Damage to Property "as the [latter]
requires proof of an additional fact which the other does not."

The two charges against petitioner, arising from the same facts, were prosecuted under the same provision
of the Revised Penal Code, as amended, namely, Article 365 defining and penalizing quasi-offenses.

The provisions contained in this article shall not be applicable. Indeed, the notion that quasi-offenses,
whether reckless or simple, are distinct species of crime, separately defined and penalized under the framework of
our penal laws, is nothing new.

The doctrine that reckless imprudence under Article 365 is a single quasi-offense by itself and not merely a
means to commit other crimes such that conviction or acquittal of such quasi-offense bars subsequent prosecution
for the same quasi-offense, regardless of its various resulting acts, undergirded this Courts unbroken chain of
jurisprudence on double jeopardy as applied to Article 365.

These cases uniformly barred the second prosecutions as constitutionally impermissible under the Double
Jeopardy Clause.
Our ruling today secures for the accused facing an Article 365 charge a stronger and simpler protection of
their constitutional right under the Double Jeopardy Clause. True, they are thereby denied the beneficent effect of
the favorable sentencing formula under Article 48, but any disadvantage thus caused is more than compensated by
the certainty of non-prosecution for quasi-crime effects qualifying as "light offenses" (or, as here, for the more
serious consequence prosecuted belatedly). If it is so minded, Congress can re-craft Article 365 by extending to
quasi-crimes the sentencing formula of Article 48 so that only the most severe penalty shall be imposed under a
single prosecution of all resulting acts, whether penalized as grave, less grave or light offenses. This will still keep
intact the distinct concept of quasi-offenses. Meanwhile, the lenient schedule of penalties under Article 365,
befitting crimes occupying a lower rung of culpability, should cushion the effect of this ruling.

Petition granted.

Criminal Law: People v Sabalones


294 SCRA 751, August 31, 1998
PEOPLE OF THE PHILIPPINES, plaintiff-appellee,
vs.ROLUSAPE SABALONES alias "Roling," ARTEMIO TIMOTEO BERONGA, TEODULO ALEGARBES and
EUFEMIO CABANERO, accused, ROLUSAPE SABALONES alias "Roling" and ARTEMIO TIMOTEO BERONGA,
accused-appellants.

Fact: Beronga, Sabalones, Alegarbes, and Cabanero were convicted after a shooting incident in Cebu in
1985 which led to the death of Glenn Tiempo and Alfredo Nardo, and fatal injuries of Nelson Tiempo, Rey
Bolo and Rogelio Presores. The victims were asked to bring the car of a certain Stephen Lim who also
attended a wedding party. Nelson Tiempo drove the car with Rogelio Presores. Alfredo Nardo drove the
owner-type jeep along with Glenn Tiempo and Rey Bolo to aid the group back to the party after parking
the car at Lims house. When they reached the gate, they were met with a sudden burst of gunfire. The
accused were identified as the gunmen. The Court of Appeals affirmed the decision of the trial court.
Sabalones and Beronga appealed.

Crime Committed: Two counts of murder, and three counts of frustrated murder

Contention of the People: Prosecution witnesses Edwin Santos and Rogelio Presores testified about the
shooting and identified the faces of the accused. Presores was riding in the car that is behind the jeep.
He positively identified Sabalones as one of the gunmen. When the gunmen fired at the car, driver Nelson
Tiempo immediately maneuvered and arrived at Major Juan Tiempos house from which they have
escaped death.

Contention of the Accused: Accused-appellants Sabalones and Beronga denied their presence during the
commission of the crime. Sabalones presented numerous witnesses who stated that he was sound asleep
when the incident took place [since he got tired watching over his brothers wake]. While Beronga
testified that he attended a cock-derby in Cebu, and was fetched by his wife at 7 pm, arrived home by
10:30 pm to sleep. Sabalones even escaped from place to place to flee from the wrath of Maj. Juan
Tiempo, the father of the two victims. The defense even pointed out errors from the testimonies of the
witnesses arguing that the place where the incident happened is dim and not lighted.

RULING: The appeal is DENIED. Costs against appellants.

Issue 1: Whether the prosecution witnesses and evidences are credible?


Yes. RTC findings were binding to court with appreciated testimonies of two witnesses. There was
positive identification by survivors who saw them when they peered during lulls in gunfire. The place was
well-lit, whether from post of cars headlights. The extrajudicial confession has no bearing because the
conviction was based on positive identification. It is binding though to the co-accused because it is used
as cirmustancial evidence corroborated by one witness. The inconcistencies are minor and
inconsequential which strengthen credibility of testimony. Furthermore, in aberratio ictus [mistake in
blow], mistake does not diminish culpability; same gravity applies, more proper to use error in personae.
Alibi cannot prevail over positive identification by the prosecution witnesses.

Issue 2: Whether the alibis are acceptable?


No. It was still quite near the crime scene. It is overruled by positive identification. Using the case of
People v. Nescio, Alibi is not credible when the accused-appellant is only a short distance from the scene
of the crime. Furthermore, flight indicates guilt.

Issue 3:Whether the correct penalty is imposed?


No. Under Article 248 of the RPC, the imposable penalty is reclusion temporal in its maximum period, to
death. There being no aggravating or mitigating circumstance, aside from the qualifying circumstance of
treachery, the appellate court correctly imposed reclusion perpetua for murder. The CA erred in
computing the penalty for each of the three counts of frustrated murder. Under Article 50 of the RPC, the
penalty for frustrated felony is next lower in degree than that prescribed by law for the consummated
felony. Because there are no mitigating or aggravating conspiracy between the two accused. It does not
matter that the prosecution has failed to show who was between the two who actually pulled the trigger
that killed the child. They are liable as co-conspirators since the act of a conspirator becomes the act of
another regardless of the precise degree of participation in the act.

Also there was a presence of treachery, because of the circumstances that the crime was done at night
time and that the accused hid themselves among the bamboo. Evident premeditation is also an
aggravating circumstance [the accused had planned to kill the victim some days before].

Bataclan v. Medina
[G.R. No. L-10126, October 22, 1957]
MONTEMAYOR, J.

Facts:
At about 2:00am of September 13, 1952, the bus, operated by its owner defendant Mariano
Medina and driven by its regular chauffeur, Conrado Saylon, left the town of Amadeo, Cavite.
While on its way to Pasay City, one of the front tires burst and the vehicle began to zig-zag
until it fell into a canal or ditch on the right side of the road and turned turtle.
Some of the passengers managed to leave the bus but the three passengers seated beside
the driver, named Bataclan, Lara and the Visayan and the woman behind them named Natalia
Villanueva, could not get out of the overturned bus. No evidence to show that the freed
passengers, including the driver and the conductor, made any attempt to pull out or extricate
and rescue the four passengers trapped inside the vehicle.
After half an hour, came about ten men, one of them carrying a lighted torch, approach the
overturned bus, and almost immediately, a fierce fire started, burning and all but consuming
the bus, including the four passengers trapped inside it.
That same day, the charred bodies of the four passengers inside the bus were removed and
duly identified that of Juan Bataclan. By reason of his death, his widow, Salud Villanueva, in
her name and in behalf of her five minor children, brought the present suit to recover from
Mariano Medina compensatory, moral, and exemplary damages and attorney's fees in the
total amount of P87,150.
After trial, the CFI Cavite awarded P1,000 to the plaintiffs plus P600 as attorney's fee, plus
P100, the value of the merchandise being carried by Bataclan to Pasay City for sale and
which was lost in the fire. Both plaintiffs and defendants appealed the case to CA which
endorsed the case to SC.
Issue: W/N the proximate cause of the death of Bataclan was the overturning of the bus or the fire that
burned the bus, including the 4 passengers left inside.

Held:
The Court held that the proximate cause was the overturning of the bus because when the
vehicle turned not only on its side but completely on its back, the leaking of the gasoline from
the tank was not unnatural or unexpected.
The coming of the men with a lighted torch was in response to the call for help, made not only
by the passengers, but most probably, by the driver and the conductor themselves, and that
because it was dark (about 2:30 in the morning), the rescuers had to carry a light with them,
and coming as they did from a rural area where lanterns and flashlights were not available.
In other words, the coming of the men with a torch was to be expected and was a natural
sequence of the overturning of the bus, the trapping of some of its passengers and the call for
outside help.
Moreover, the burning of the bus can also in part be attributed to the negligence of the carrier,
through its driver and its conductor. According to the witness, the driver and the conductor
were on the road walking back and forth. They, or at least, the driver should and must have
known that in the position in which the overturned bus was, gasoline could and must have
leaked from the gasoline tank and soaked the area in and around the bus.
The leaked gasoline can be smelt and directed even from a distance, and yet neither the
driver nor the conductor would appear to have cautioned or taken steps to warn the rescuers
not to bring the lighted torch too near the bus.
In addition, the case involves a breach of contract of transportation because the Medina
Transportation failed to carry Bataclan safely to his destination, Pasay City. There was
likewise negligence on the part of the defendant, through his agent, the driver Saylon. There is
evidence to show that at the time of the blow out, the bus was speeding and that the driver
failed to changed the tires into new ones as instructed by Mariano Medina.
The driver had not been diligent and had not taken the necessary precautions to insure the
safety of his passengers. Had he changed the tires, specially those in front, with new ones, as
he had been instructed to do, probably, despite his speeding, the blow out would not have
occurred.
Ratio:
Proximate cause is that cause, which, in natural and continuous sequence, unbroken by any
efficient intervening cause, produces the injury, and without which the result would not have
occurred.
Comprehensively, 'the proximate legal cause is that acting first and producing the injury, either
immediately or by setting other events in motion, all constituting a natural and continuous
chain of events, each having a close causal connection with its immediate predecessor, the
final event in the chain immediately effecting the injury as a natural and probable result of the
cause which first acted, under such circumstances that the person responsible for the first
event should, as an ordinary prudent and intelligent person, have reasonable ground to expect
at the moment of his act or default that an injury to some person might probably result
therefrom.

Urbano v. IAC
Facts:
On October 23, 1980, petitioner Filomeno Urbano was on his way to his
ricefield. He found the place where he stored palay flooded with water coming
from the irrigation canal. Urbano went to the elevated portion to see what
happened, and there he saw Marcelino Javier and Emilio Efre cutting grass.
Javier admitted that he was the one who opened the canal. A quarrel ensued,
and Urbano hit Javier on the right palm with his bolo, and again on the leg
with the back of the bolo. On October 27, 1980, Urbano and Javier had an
amicable settlement. Urbano paid P700 for the medical expenses of Javier. On
November 14, 1980, Urbano was rushed to the hospital where he had lockjaw
and convulsions. The doctor found the condition to be caused by tetanus toxin
which infected the healing wound in his palm. He died the following day.
Urbano was charged with homicide and was found guilty both by the trial
court and on appeal by the Court of Appeals. Urbano filed a motion for new
trial based on the affidavit of the Barangay Captain who stated that he saw the
deceased catching fish in the shallow irrigation canals on November 5. The
motion was denied; hence, this petition.
Issue:
Whether the wound inflicted by Urbano to Javier was the proximate cause of
the latters death
Held:
A satisfactory definition of proximate cause is... "that cause, which, in natural
and continuous sequence, unbroken by any efficient intervening cause,
produces the injury, and without which the result would not have
occurred."And more comprehensively, "the proximate legal cause is that
acting first and producing the injury, either immediately or by setting other
events in motion, all constituting a natural and continuous chain of events,
each having a close causal connection with its immediate predecessor, the
final event in the chain immediately effecting the injury as a natural and
probable result of the cause which first acted, under such circumstances that
the person responsible for the first event should, as an ordinarily prudent and
intelligent person, have reasonable ground to expect at the moment of his act
or default that an injury to some person might probably result therefrom."
If the wound of Javier inflicted by the appellant was already infected by
tetanus germs at the time, it is more medically probable that Javier should
have been infected with only a mild cause of tetanus because the symptoms of
tetanus appeared on the 22nd dayafterthe hacking incident ormore than 14
days after the infliction of the wound. Therefore, theonset time should have
been more than six days. Javier, however, died on the second day from
theonset time. The more credible conclusion is that at the time Javier's wound
was inflicted by the appellant, the severe form of tetanus that killed him was
not yet present. Consequently, Javier's wound could have been infected with
tetanus after the hacking incident. Considering the circumstance surrounding
Javier's death, his wound could have been infected by tetanus 2 or 3 or a few
but not 20 to 22 days before he died.
The rule is that the death of the victim must be thedirect, natural, and logical
consequence of the wounds inflicted upon him by the accused. And since we
are dealing with a criminal conviction, the proof that the accused caused the
victim's death must convince a rational mind beyond reasonable doubt. The
medical findings, however, lead us to a distinct possibility that the infection of
the wound by tetanus was an efficient intervening cause later or between the
time Javier was wounded to the time of his death. The infection was,
therefore, distinct and foreign to the crime.
There is a likelihood that the wound was but the remote cause and its
subsequent infection, for failure to take necessary precautions, with tetanus
may have been the proximatecause of Javier's death with which the petitioner
had nothing to do. "A prior and remote cause cannot be made the be of an
action if such remote cause did nothing more than furnish the condition or
give rise to the occasion by which the injury was made possible, if there
intervened between such prior or remote cause and the injury a distinct,
successive, unrelated, and efficient cause of the injury, even though such
injury would not have happened but for such condition or occasion. If no
danger existed in the condition except because of the independent cause, such
condition was not the proximate cause. And if an independent negligent act or
defective condition sets into operation the instances which result in injury
because of the prior defective condition, such subsequent act or condition is
the proximate cause."
Crim Law 1 Case Digest:Intod V. CA 1992
Laws Applicable:

FACTS:
February 4, 1979: Sulpicio Intod, Jorge Pangasian, Santos Tubio and Avelino Daligdig went to
Salvador Mandaya's house and asked him to go with them to the house of Bernardina
Palangpangan. Thereafter, they had a meeting with Aniceto Dumalagan who told Mandaya that he
wanted Palangpangan to be killed because of a land dispute between them and that Mandaya
should accompany them. Otherwise, he would also be killed.
February 4, 1979 10:00 pm: All of them armed arrived at Palangpangan's house and fired at
Palangpangan's bedroom but there was no one in the room.
RTC: convicted Intod of attempted murder based on the testimony of the witness

ISSUE: W/N Intod is guilty attempted murder since it is an impossible crime under Art. 4 (2)

HELD: YES. petition is hereby GRANTED, the decision of respondent Court of Appeals holding
Petitioner guilty of Attempted Murder is hereby MODIFIED. sentences him to suffer the penalty of six
(6) months of arresto mayor, together with the accessory penalties provided by the law, and to pay
the costs

Art. 4(2). CRIMINAL RESPONSIBILITY. Criminal Responsibility shall be incurred:


xxx xxx xxx
2. By any person performing an act which would be an offense against persons or property, were it
not for the inherent impossibility of its accomplishment or on account of the employment of
inadequate or ineffectual means.
Petitioner contends that, Palangpangan's absence from her room on the night he and his
companions riddled it with bullets made the crime inherently impossible.
The Revised Penal Code, inspired by the Positivist School, recognizes in the offender his
formidability to punish criminal tendencies in Art. 4(2)
Legal impossibility occurs where the intended acts, even if completed, would not amount to a
crime
Legal impossibility would apply to those circumstances where
1. the motive, desire and expectation is to perform an act in violation of the law
2. there is intention to perform the physical act
3. there is a performance of the intended physical act
4. the consequence resulting from the intended act does not amount to a crime
o Ex: The impossibility of killing a person already dead
Factual impossibility occurs when extraneous circumstances unknown to the actor or beyond his
control prevent the consummation of the intended crime this case
o Ex: man who puts his hand in the coat pocket of another with the intention to steal the latter's
wallet and finds the pocket empty
United States: where the offense sought to be committed is factually impossible or
accomplishment - attempt to commit a crime; legally impossible of accomplishment - cannot be held
liable for any crime

JACINTO vs PEOPLE

GEMMA JACINTO vs PEOPLE


G.R. NO. 162540 13July2009 592SCRA26

FACTS: In June 1997, Baby Aquino, handed petitioner -collector of Mega Foam, a post dated checked
worth P10,000 as payment for Babys purchases from Mega Foam International, Inc. The said check was
deposited to the account of Jacqueline Capitles husband-Generoso. Rowena Recablanca, another
employee of Mega Foam, received a phone call from an employee of Land Bank, who was looking for
Generoso to inform Capitle that the BDO check deposited had been dishonored. Thereafter, Joseph
Dyhenga talked to Baby to tell that the BDO Check bounced. However, Baby said that she had already
paid Mega Foam P10,000 cash in August 1997 as replacement for the dishonored check.
Dyhengco filed a compliant with the National Bureau of Investigation (NBI) and worked out an
entrapment operation with its agents. Thereafter, petitioner and Valencia were arrested. The NBI filed a
criminal case for qualified theft against the two (2) and Jacqueline Capitle.

RTC rendered a decision that Gemma, Anita and Jacqueline GUILTY beyond reasonable doubt of the
crime of QUALIFIED THEFT and each of the sentenced to suffer imprisonment of Five (5) years, Five (5)
months and Eleven (11) days to Six (6) years, Eight (8) months and Twenty (20) days.

ISSUE:Whether or not the crime committed falls the definition of Impossible Crime.
HELD:Yes, Since the crime of theft is not a continuing offense, petitioners act of receiving the cash
replacement should not be considered as continuation of the Theft.
The requisites of an impossible crime are:

1. That the Act performed would be an offer against persons or property;


2. That the act was alone with evil intent; and
3. That the accomplishment was inherently impossible or the means employed was either inadequate or
ineffectual.
The time that petitioner took a possession of the check meant for Mega Foam, she had performed all the
acts to consummate that crime of theft had it not been impossible of accomplishment in this case.

Therefore, the Supreme Court held that petitioner Gemma T. Jacinto is foundGUILTY of an
impossible crimeand suffer the penalty of Six (6) months of arresto mayor and pay courts.
US v. Eduave (36 Phil. 209, February
2, 1917)
FACTS:

Defendant Protasio Eduave is the querido of the victims mother. Eduave attacked the victim
from behind using a bolo creating a gash 8 1/2 inches long and 2 inches deep because the latter
accused defendant of having commiting rape against said victim. Upon thinking that he has
already killed the victim, he threw the body into the bushes and left.

ISSUE/S:

What is the crime committed by Eduave?

HELD:

Accused is guilty of frustrated murder. The fact that Eduave attacked the victim from behind, in a
vital portion of the body, shows treachery qualifying it as murder. The crime was not
consummated because the elements of the crimes execution and accomplishment were not
complete as the victim did not die. Neither was the crime an attempted one because the accuseds
actions has already passed the subjective phase, that is, there was no external force preventing
defendant from performing all the acts of execution necessary to commit the felony.
Consequently, the victim did not die because an external element has prevented such death after
Eduave has performed all the necessary acts of execution that would have caused the death of the
victim

MORELAND, J.:
We believe that the accused is guilty of frustrated murder.
We are satisfied that there was an intent to kill in this case. A deadly weapon was used. The blow was
directed toward a vital part of the body. The aggressor stated his purpose to kill, thought he had killed,
and threw the body into the bushes. When he gave himself up he declared that he had killed the
complainant.
There wasalevosiato qualify the crime as murder if death had resulted. The accused rushed upon the girl
suddenly and struck her from behind, in part at least, with a sharp bolo, producing a frightful gash in the
lumbar region and slightly to the side eight and one-half inches long and two inches deep, severing all of
the muscles and tissues of that part.
The motive of the crime was that the accused was incensed at the girl for the reason that she had
theretofore charged him criminally before the local officials with having raped her and with being the
cause of her pregnancy. He was her mother'squeridoand was living with her as such at the time the crime
here charged was committed.
That the accused is guilty of some crime is not denied. The only question is the precise crime of which he
should be convicted. It is contended, in the first place, that, if death had resulted, the crime would not
have been murder but homicide, and in the second place, that is is attempted and not frustrated homicide.
As to the first contention, we are of the opinion that the crime committed would have been murder if the
girl had been killed. It is qualified by the circumstance ofalevosia, the accused making a sudden attack
upon his victim from the rear, or partly from the rear, and dealing her a terrible blow in the back and side
with his bolo. Such an attack necessitates the finding that it was made treacherously' and that being so the
crime would have been qualified as murder if death had resulted.
As to the second contention, we are of the opinion that the crime was frustrated and not attempted
murder. Article 3 of the Penal Code defines a frustrated felony as follows:
"A felony is frustrated when the offender performs all the acts of execution which should produce the
felony as a consequence, but which, nevertheless, do not produce it by reason of causes independent of the
will of the perpetrator."
An attempted felony is defined thus:
"There is an attempt when the offender commences the commission of the felony directly by overt acts,
and does not perform all the acts of execution which constitute the felony be reason of some cause or
accident other than his own voluntarily desistance."
The crime cannot be attempted murder. This is clear from the fact that the defendant performedallof the
acts which should have resulted in the consummated crime andvoluntarilydesisted from further acts. A
crime cannot be held to be attempted unless the offender, after beginning the commission of the crime by
overt acts, is prevented, against his will, by some outside cause from performing all of the acts which
should produce the crime. In other words, to be an attempted crime the purpose of the offender must be
thwarted by a foreign force or agency which intervenes and compels him to stop prior to the moment
when he has performed all of the acts which should produce the crime as a consequence, which acts it is
his intention to perform. If he has performed all of the acts which should result in the consummation of
the crime andvoluntarilydesists from proceeding further, it can not be an attempt. The essential element
which distinguishes attempted form frustrated felony is that, in the latter, there is no intervention of a
foreign or extraneous cause or agency between the beginning of the commission of the crime and the
moment when all of the acts have been performed which should result int he consummated crime; while
in the former there is such intervention and the offender does not arrive at the point of performingallof
the acts which should produce the crime. He is stopped short of that point by some cause apart from his
voluntary desistance.
To put it in another way, in case of an 'attempt the offender never passes the subjective phase of he
offense. He is interrupted and compelled to desist by the intervention of outside causes before the
subjective phase is passed.
On the other had, in case of frustrated crimes the subjective phase is completely passed. Subjectively the
crime is complete. Nothing interrupted the offender while he was passing through the subjective phase.
The crime, however, is not consummated by reason of the intervention of causes independent of the will
of the offender. He did all that was necessary to commit the crime. If the crime did not result as a
consequence it was due to something beyond his control.
The subjective phase is that portion of the acts constituting the crime included between the act
whichbeginsthe commission of the crime and thelastact performed by the offender which, with the
prior acts, should result in the consummated crime. From that time forward the phase is objective. It may
also be said to be that period occupied by the acts of the offender over which he has control that period
between the point where hebeginsand the point where the point where hevoluntarily desists. Ifbetween
these two pointsthe offender is stopped by reason of any cause outside of his own voluntary desistance,
the subjective phase has not been passed and it is an attempt. If he is not so stopped but continues until
he performs the last act, it is frustrated.
That the case before us is frustrated is clear.
The penalty should have been thirteen years ofcadena temporalthere being neither aggravating nor
mitigating circumstance. As so modified, the judgment is affirmed with costs. So ordered.

Petitioners: Esmeraldo Rivera, Ismael Rivera, and Edgardo Rivera


Respondent: People of the Philippines
Ponente: J. Callejo, Sr.

FACTS:
As the victim, Ruben Rodil, went to a nearby store to buy food, accused Edgardo Rivera mocked
him for being jobless and dependent on his wife for support. Ruben resented the rebuke and
thereafter, a heated exchange of words ensued. In the evening of the following day, when Ruben
and his three-year-old daughter went to the store to buy food, Edgardo, together with his brother
Esmeraldo Rivera and Ismael Rivera, emerged from their house and ganged up on
him. Esmeraldo and Ismael mauled Ruben with fist blows. And as he fell to the ground, Edgardo
hit him three times with a hollow block on the parietal area. Esmeraldo, Ismael and Edgardo fled
to their house only when the policemen arrived. Ruben sustained injuries and was brought to the
hospital. The doctor declared that the wounds were slight and superficial, though the victim
could have been killed had the police not promptly intervened. The trial court found the accused
guilty of the crime of frustrated murder. An appeal was made by the accused, but the Court of
Appeals affirmed the trial courts decision with modification, changing the crime to attempted
murder and imposed an indeterminate penalty of 2 years of prision correccional as minimum to 6
years and 1 day of prision mayor as maximum.

ISSUES:
1) Whether or not there was intent to kill.
2) Whether or not the Court of Appeals was correct in modifying the crime from frustrated to
attempted murder.
3) Whether or not the aggravating circumstance of treachery was properly applied.
4) Whether or not the correct penalty was imposed.

HELD:
1) Yes. The Court declared that evidence to prove intent to kill in crimes against persons may
consist, inter alia, in the means used by the malefactors, the nature, location and number of
wounds sustained by the victim, the conduct of the malefactors before, at the time, or
immediately after the killing of the victim, the circumstances under which the crime was
committed and the motives of the accused. In the present case, Esmeraldo and Ismael pummeled
the victim with fist blows, while Edgardo hit him three times with a hollow block. Even though
the wounds sustained by the victim were merely superficial and could not have produced his
death, intent to kill was presumed.

2) Yes. Article 6 of the Revised Penal Code provides that there is an attempt when the offender
commences the commission of a felony directly by overt acts, and does not perform all the acts
of execution which should produce the felony by reason of some cause or accident other than his
own spontaneous desistance. Although the wounds sustained by the victim were merely
superficial and could not have produced his death, it does not negate criminal liability of the
accused for attempted murder. The intent to kill was already presumed based on the overt acts of
the accused. In fact, victim could have been killed had the police not promptly intervened.

3) Yes. The essence of treachery is the sudden and unexpected attack, which gives no opportunity
for the victim to repel it or defend himself. In the present case, the accused attacked the victim in
a sudden and unexpected manner as he was walking with his three-year-old daughter, impervious
of the imminent peril to his life. He was overwhelmed with the assault of the accused and had no
chance to defend himself and retaliate. Thus, there was treachery.

4) No. Under Article 248 of the Revised Penal Code, as amended by Republic Act No. 7659, the
penalty for murder is reclusion perpetua to death. Since the accused were guilty only of
attempted murder, the penalty should be reduced by two degrees, in accordance to Article 51 of
the Revised Penal Code. Thus, under Article 61 (2), in relation to Article 71 of the Revised Penal
Code, the penalty should be prision mayor. In the absence of any modifying circumstance in the
commission of the crime other than the qualifying circumstance of treachery, the maximum of
the indeterminate penalty shall be taken from the medium period of prision mayor which has a
range of from eight (8) years and one (1) day to ten (10) years. To determine the minimum of the
indeterminate penalty, the penalty of prision mayor should be reduced by one degree, prision
correccional, which has a range of six (6) months and one (1) day to six (6) years. Hence, the
accused were sentenced to suffer an indeterminate penalty of from two (2) years of prision
correccional in its minimum period, as minimum, to nine (9) years and four (4) months of prision
mayor in its medium period, as maximum.

Crim Law 1 Case Digest: Baleros V. People


2006
FACTS:
Martina Lourdes Albano (Malou), a medical student of the University of Sto. Tomas, stayed at
Room 307 with her maid Marvilou.
December 12 10:30 pm: Malou slept. Her maid Marvilou slept on a folding bed right in front of
her bedroom door.
December 13, 1991 1:00 am: Chito left the fraternity party with Robert Chan and Alberto wearing
a barong tagalog, with t-shirt inside, with short pants with stripes lent by Perla Duran and leather
shoes.
December 13, 1991 1:30 am: Chito arrived at the Building wearing a white t-shirt with fraternity
symbols and black shorts with the brand name Adidas from a party. He requested permission from
S/G Ferolin to go up to Room 306 leased by Ansbert Co but at that time only Joseph Bernard Africa
was there. Although Chito could not produce the required written authorization, he let him in
because he will be a tenant in the coming summer break. Joseph was awaken by Chitos knock so
he glanced the alarm clock and let him. He saw him wearing dark-colored shorts and white T-shirt.
December 13, 1991 1:50 am: Renato Baleros, Jr. y David (Chito) forcefully covered the face of
Martina Lourdes T. Albano with a piece of cloth soaked in chemical with dizzying effects. This
awakened Malou. She struggled but could not move because she was tightly held and pinned down
on the bed. She kicked him and got her right hand free to squeeze his sex organ causing him to let
her go. She went for the bedroom door and woke up Marvilou. She also intercommed S/G Ferolin
saying: "may pumasok sa kuarto ko pinagtangkaan ako". Malou proceed to Room 310 where her
classmates Christian Alcala, Bernard Baptista, Lutgardo Acosta and Rommel Montes were staying
and seeked help. She saw her bed in a mess and noticed that her nightdress was stained with
blue. Aside from the window with grills which she had originally left opened, another window inside
her bedroom which leads to Room 306 was now open.
December 13, 1991 3:30 pm: Christian and his roommates, Bernard and Lutgardo were asked by
the CIS people to look for anything not belonging to them in their Unit when Rommel Montes went
inside and found a grey bag.
o Christian knew right away that it belonged to Chito. It contained white t-shirt with fraternity
symbol, a Black Adidas short pants, a handkerchief , 3 white T-shirts, an underwear and socks.
Chito pleaded NOT Guilty
13 witnesses including Malou and her classmates, Joseph Bernard Africa, Rommel Montes,
Renato Alagadan and Christian Alcala
o Malou: Chito was her classmate whom he rejected a week before
o Chito: He only slept and at about 6 to 6:30, Joseph told him that something had happened and
asked him to follow him to Room 310 carrying his gray bag and since no one was there they went to
Room 401 where Renato Alagadan was. He left his grey bag at Room 306 the day before.
handkerchief and Malous night dress both contained chloroform, a volatile poison which causes
first degree burn exactly like what Malou sustained on that part of her face where the chemical-
soaked cloth had been pressed
RTC: guilty of attempted rape
CA: Affirmed

ISSUE: W/N Chito is guilty of attempted rape

HELD: NO. REVERSED and SET ASIDE. ACQUITTING Renato D. Baleros, Jr. of the charge for
attempted rape. GUILTY of light coercion and is accordingly sentenced to 30 days of arresto menor
and to pay a fine of P200.00, with the accessory penalties thereof and to pay the costs.

Under Article 335 of the Revised Penal Code, rape is committed by a man who has carnal
knowledge or intercourse with a woman under any of the following circumstances: (1) By using force
or intimidation; (2) When the woman is deprived of reason or otherwise unconscious; and (3) When
the woman is under twelve years of age or is demented.
Under Article 6, in relation to the aforementioned article of the same code, rape is attempted
when the offender commences the commission of rape directly by overt acts and does not perform
all the acts of execution which should produce the crime of rape by reason of some cause or
accident other than his own spontaneous desistance.
o whether or not the act of the petitioner, i.e., the pressing of a chemical-soaked cloth while on top
of Malou, constitutes an overt act of rape.
o Overt or external act has been defined as some physical activity or deed, indicating the intention
to commit a particular crime, more than a mere planning or preparation, which if carried out to its
complete termination following its natural course, without being frustrated by external obstacles nor
by the voluntary desistance of the perpetrator, will logically and necessarily ripen into a concrete
offense
Chito was fully clothed and that there was no attempt on his part to undress Malou, let alone
touch her private part
Verily, while the series of acts committed by the petitioner do not determine attempted rape, they
constitute unjust vexation punishable as light coercion under the second paragraph of Article 287 of
the Revised Penal Code.
o As it were, unjust vexation exists even without the element of restraint or compulsion for the
reason that this term is broad enough to include any human conduct which, although not productive
of some physical or material harm, would unjustly annoy or irritate an innocent person
o That Malou, after the incident in question, cried while relating to her classmates what she
perceived to be a sexual attack and the fact that she filed a case for attempted rape proved beyond
cavil that she was disturbed, if not distressed

Crim Law 1 Case Digest: Valenzuela V. People


2007
FACTS:
May 19, 1994 4:30 pm: Aristotel Valenzuela and Jovy Calderon were sighted outside the Super
Sale Club, a supermarket within the ShoeMart (SM) complex along North EDSA, by Lorenzo Lago, a
security guard who was then manning his post at the open parking area of the supermarket. Lago
saw Valenzuela, who was wearing an ID with the mark Receiving Dispatching Unit (RDU) who
hauled a push cart with cases of detergent of Tide brand and unloaded them in an open parking
space, where Calderon was waiting. He then returned inside the supermarket and emerged 5
minutes after with more cartons of Tide Ultramatic and again unloaded these boxes to the same area
in the open parking space. Thereafter, he left the parking area and haled a taxi. He boarded the cab
and directed it towards the parking space where Calderon was waiting. Calderon loaded the cartons
of Tide Ultramatic inside the taxi, then boarded the vehicle. As Lago watched, he proceeded to stop
the taxi as it was leaving the open parking area and asked Valenzuela for a receipt of the
merchandise but Valenzuela and Calderon reacted by fleeing on foot. Lago fired a warning shot to
alert his fellow security guards. Valenzuela and Calderon were apprehended at the scene and the
stolen merchandise recovered worth P12,090.
Valenzuela, Calderon and 4 other persons were first brought to the SM security office before they
were transferred to the Baler Station II of the Philippine National Police but only Valenzuela and
Calderon were charged with theft by the Assistant City Prosecutor.
They pleaded not guilty.
Calderons Alibi: On the afternoon of the incident, he was at the Super Sale Club to withdraw
from his ATM account, accompanied by his neighbor, Leoncio Rosulada. As the queue for the ATM
was long, he and Rosulada decided to buy snacks inside the supermarket. While they were eating,
they heard the gunshot fired by Lago, so they went out to check what was transpiring and when
they did, they were suddenly grabbed by a security guard
Valenzuelas Alibi: He is employed as a bundler of GMS Marketing and assigned at the
supermarket. He and his cousin, a Gregorio Valenzuela, had been at the parking lot, walking beside
the nearby BLISS complex and headed to ride a tricycle going to Pag-asa, when they saw the
security guard Lago fire a shot causing evryon to start running. Then they were apprehended by
Lago.
RTC: guilty of consummated theft
CA: Confirmed RTC and rejected his contention that it should only be frustrated theft since at the
time he was apprehended, he was never placed in a position to freely dispose of the articles stolen.

ISSUE: W/N Valenzuela should be guilty of consummated theft.

HELD: YES. petition is DENIED


Article 6 defines those three stages, namely the consummated, frustrated and attempted
felonies.
o A felony is consummated when all the elements necessary for its execution and
accomplishment are present.
o It is frustrated when the offender performs all the acts of execution which would produce the
felony as a consequence but which, nevertheless, do not produce it by reason of causes
independent of the will of the perpetrator.
o It is attempted when the offender commences the commission of a felony directly by overt acts,
and does not perform all the acts of execution which should produce the felony by reason of some
cause or accident other than his own spontaneous desistance.
Each felony under the Revised Penal Code has a:
o subjective phase - portion of the acts constituting the crime included between the act which
begins the commission of the crime and the last act performed by the offender which, with prior acts,
should result in the consummated crime
if the offender never passes the subjective phase of the offense, the crime is merely attempted
o objective phase - After that point of subjective phase has been breached
subjective phase is completely passed in case of frustrated crimes
the determination of whether a crime is frustrated or consummated necessitates an initial
concession that all of the acts of execution have been performed by the offender
The determination of whether the felony was produced after all the acts of execution had been
performed hinges on the particular statutory definition of the felony.
actus non facit reum, nisi mens sit rea - ordinarily, evil intent must unite with an unlawful act for
there to be a crime or there can be no crime when the criminal mind is wanting
In crimes mala in se, mens rea has been defined before as a guilty mind, a guilty or wrongful
purpose or criminal intent and essential for criminal liability.
Statutory definition of our mala in se crimes must be able to supply what the mens rea of the
crime is and overt acts that constitute the crime
Article 308 of the Revised Penal Code (Elements of Theft):
1. that there be taking of personal property - only one operative act of execution by the actor
involved in theft
2. property belongs to another
3. taking be done with intent to gain - descriptive circumstances
4. taking be done without the consent of the owner - descriptive circumstances
5. taking be accomplished without the use of violence against or intimidation of persons or force
upon things - descriptive circumstances
Abandoned cases:
o U.S. v. Adiao: failed to get the merchandise out of the Custom House - consummated theft
o Dio: Military Police inspected the truck at the check point and found 3 boxes of army rifles -
frustrated theft
o Flores: guards discovered that the empty sea van had actually contained other merchandise as
well - consummated theft
o Empelis v. IAC: Fled the scene, dropping the coconuts they had seized - frustrated qualified theft
because petitioners were not able to perform all the acts of execution which should have produced
the felony as a consequence
cannot attribute weight because definition is attempted
The ability of the actor to freely dispose of the articles stolen, even if it were only momentary.
o We are satisfied beyond reasonable doubt that the taking by the petitioner was completed in this
case. With intent to gain, he acquired physical possession of the stolen cases of detergent for a
considerable period of time that he was able to drop these off at a spot in the parking lot, and long
enough to load these onto a taxicab.
Article 308 of the Revised Penal Code, theft cannot have a frustrated stage. Theft can only be
attempted (no unlawful taking) or consummated (there is unlawful taking).

People vs Lamahang Digest


Posted on August 15, 2016

PEOPLE of the PHILIPPINES vs LAMAHANG

FACTS:

The defendant Aurelio Lamahang is on appeal from a decision finding him guilty of attempted robbe
ry.
At early dawn on March 2, 1935, policeman Jose Tomambing, who was patrolling his beat on Delgado
and C.R. Fuentes streets of the City of Iloilo, caughtthe accused in the act of making an opening wi
th an iron bar on the wall of a store of cheap goods located on the last named street.
At that time the owner of the store, Tan Yu, was sleeping inside with another Chinaman.
The accusedhad only succeeded in breaking one board and in unfastening another from the wal
l, when the policeman showed up, who instantly arrested him and placed him under custody.
ISSUE:

WON the accused was erroneously declared guilty of attempted robbery

RULING:

YES, he was erroneously declared guilty of attempted robbery. The accused is then held guilty of attempted tre
spass to dwelling, committed by means of force, with the aforesaid aggravating and mitigating circumstances a
nd sentenced to three months and one day of arresto mayor.

RATIONALE:

It is necessary to prove that said beginning of execution, if carried to its complete termination following its natur
al course, without being frustrated by external obstacles nor by the voluntary desistance of the perpetrator, will l
ogically and necessarily ripen into a concrete offense. In the case of robbery, it must be shown that the offen
der clearly intended to take possession, for the purpose of gain, of some personal property belonging t
o another. In the instant case, it may only be inferred as a logical conclusion that his evident intention was to e
nter by means of force said store against the will of its owner. That his final objective, once he succeeded in ent
ering the store, was to rob, to cause physical injury to the inmates, or to commit any other offense, there is noth
ing in the record to justify a concrete finding.

It must be borne in mind (I Groizard, p. 99) that in offenses not consummated, as the material damage is wa
nting, the nature of the action intended (accion fin) cannot exactly be ascertained, but the same must be i
nferred from the nature of the acts executed (accion medio). The relation existing between the facts submi
tted for appreciation and the offense which said facts are supposed to produce must be direct; the intention mu
st be ascertained from the facts and therefore it is necessary, in order to avoid regrettable instances of injustice
.

Under article 280 of the Revised Penal Code, the Court is of the opinion that the fact under consideration does
not constitute attempted robbery but attempted trespass to dwelling. Against the accused must be taken into c
onsideration the aggravating circumstances of nighttime and former convictions, inasmuch as the record sho
ws that several final judgments for robbery and theft have been rendered against him and in his favor, the mi
tigating circumstance of lack of instruction.

=

Judgment appealed is affirmed.
Criminal Law: People v Orita
184 SCRA 105, April 3, 1990
THE PEOPLE OF THE PHILIPPINES, plaintiff-appellee,
vs.CEILITO ORITA alias "Lito," defendant-appellant

Facts: Ceilito Orita was accused of frustrated rape by the RTC. He appealed to the Court of Appeals for
review. The accused poke a balisong to college freshman Cristina Abayan as soon as she got into her
boarding house early morning after arriving from a party. She knew him as a frequent visitor of another
boarder. She was dragged inside the house up the stairs while his left arm wrapped around her neck, and
his right hand poking the Batangas knife to her neck. Upon entering her room, he pushed her in and got
her head hit on the wall. He immediately undressed while still holding the knife with one hand, and
ordered her to do the same. He ordered her to lie down on the floor and then mounted her. He asked her
to hold his penis and insert it in her vagina, while still poking the knife to her. She followed, but the
appellant could not fully penetrate her in such a position. Next, he laid down on his back and commanded
her to mount him, but he cannot fully penetrate her. When Oritas hands were both flat on the floor,
complainant escaped naked. She ran from room to room as appellant pursued her, and finally jumped out
through a window. She went to the municipal building nearby and knocked on the back door for there
was no answer. When the door opened, the policemen inside the building saw her crying and naked. She
was given a jacket for covering by the first policeman who saw her. The policemen dashed to her
boarding house but failed to apprehend the accused. She was brought to a hospital for physical
examination. Her PE revealed that she is still a virgin, with abrasions on the left breast, left and right
knees, and multiple pinpoint marks on her back, among others. The trial court convicted the accused of
frustrated rape.

Crime Committed: Frustrated Rape


Issue: Whether or not the frustrated stage applies to the crime of rape?

Contention of the Accused: The accused contends that there is no crime of frustrated rape. The trial court
erred in disregarding the substantial inconsistencies in the testimonies of the witnesses; and the trial
court erred in declaring that the crime of frustrated rape was committed by the accused. He was not able
to fully penetrate in her. The accused also questions also the failure of the prosecution to present other
witnesses to corroborate the allegations in the complaint. The accused used the Article 266 of the RPC to
show that he is not guilty of frustrated rape, and Article 6 to stress the difference of consummated,
frustrated, and attempted felonies.

Contention of the People: The victim's testimony from the time she knocked on the door of the municipal
building up to the time she was brought to the hospital was corroborated by Pat. Donceras. Rather than
discredit the testimonies of the prosecution witnesses, discrepancies on minor details must be viewed as
adding credence and veracity to such spontaneous testimonies. The accused committed rape.

Ruling: The decision of the RTC is hereby MODIFIED. The accused Ceilito Orita is hereby found guilty
beyond reasonable doubt of the crime of rape [consummated] and sentenced to reclusion perpetua as
well as to indemnify the victim in the amount of P30,000.00.

Clearly, in the crime of rape, from the moment the offender has carnal knowledge of his victim he actually
attains his purpose and, from that moment also all the essential elements of the offense have been
accomplished. Nothing more is left to be done by the offender, because he has performed the last act
necessary to produce the crime. Thus, the felony is consummated. [Art. 266 and Art. 6]
We have set the uniform rule that for the consummation of rape, perfect penetration is not essential. Any
penetration of the female organ by the male organ is sufficient. Entry of the labia or lips of the female
organ, without rupture of the hymen or laceration of the vagina is sufficient to warrant conviction

Lessons Applicable: No frustrated rape same case

Laws Applicable: Art. 6

FACTS:
March 20, 1983 Early Morning: Cristina S. Abayan, 19-year old freshman student at the St.
Joseph's College, arrived at her boarding house after her classmates brought her home from a
party. She knocked at the door of her boarding house when a frequent visitor of another boarder
held her and poked a knife to her neck. Despite pleading for her release, he ordered her to go
upstairs with him. Since the door which led to the 1st floor was locked from the inside, they used the
back door to the second floor. With his left arm wrapped around her neck and his right hand poking a
"balisong" to her neck, he dragged her up the stairs. When they reached the second floor, he
commanded herwith the knife poked at her neck, to look for a room. They entered Abayan's room.
He then pushed her hitting her head on the wall. With one hand holding the knife, he undressed
himself. He then ordered her to take off her clothes. Scared, she took off her T-shirt, bra, pants and
panty. He ordered her to lie down on the floor and then mounted her. He made her hold his penis
and insert it in her vagina. Still poked with a knife, she did as told but since she kept moving, only a
portion of his penis entered her. He then laid down on his back and commanded her to mount him.
Still only a small part of his penis was inserted into her vagina. When he had both his hands flat on
the floor. She dashed out to the next room and locked herself in. When he pursued her and climbed
the partition, she ran to another room then another then she jumped out through a window.
Still naked, she darted to the municipal building, 18 meters in front of the boarding house and
knocked on the door. When there was no answer, she ran around the building and knocked on the
back door. When the policemen who were inside the building opened the door, they found her
naked sitting on the stairs crying. Pat. Donceras, took off his jacket and wrapped it around her. Pat.
Donceras and two other policemen rushed to the boarding house where they heard and saw
somebody running away but failed to apprehend him due to darkness. She was taken to Eastern
Samar Provincial Hospital where she was physically examined.
Her vulva had no abrasions or discharges.
RTC: frustrated rape

ISSUE: W/N there is frustrated rape.

HELD: NO. RTC MODIFIED. guilty beyond reasonable doubt of the crime of rape and sentenced to
reclusion perpetua as well as to indemnify the victim in the amount of P30,000
Correlating Art. 335 and Art. 6, there is no debate that the attempted and consummated stages
apply to the crime of rape.
Requisites of a frustrated felony are:
o (1) that the offender has performed all the acts of execution which would produce the felony
o (2) that the felony is not produced due to causes independent of the perpetrator's will
attempted crime the purpose of the offender must be thwarted by a foreign force or agency which
intervenes and compels him to stop prior to the moment when he has performed all of the acts which
should produce the crime as a consequence, which acts it is his intention to perform
o If he has performed all of the acts which should result in the consummation of the crime and
voluntarily desists from proceeding further, it can not be an attempt.
in the crime of rape, from the moment the offender has carnal knowledge of his victim he actually
attains his purpose and, from that moment also all the essential elements of the offense have been
accomplished. Any penetration of the female organ by the male organ is sufficient. Entry of the labia
or lips of the female organ, without rupture of the hymen or laceration of the vagina is sufficient to
warrant conviction. Necessarily, rape is attempted if there is no penetration of the female organ
The fact is that in a prosecution for rape, the accused may be convicted even on the sole basis of
the victim's testimony if credible. Dr. Zamora did not rule out penetration of the genital organ of the
victim.

PEOPLE v. CAMPUHAN
G.R. No. 129433. March 30, 2000.
Stages of rape
In the case ofPeople v. Orita, the SC held that rape was consummated from the moment the
offender had carnal knowledge of the victim since by it he attained his objective. All the elements
of the offense were already present and nothing more was left for the offender to do, having
performed all the acts necessary to produce the crime and accomplish it. We ruled then that
perfect penetration was not essential;any penetrationof the female organ by the male organ,
however slight, was sufficient. The Court further held that entry of thelabiaor lips of the female
organ, even without rupture of the hymen or laceration of the vagina, was sufficient to warrant
conviction for consummated rape. We distinguished consummated rape from attempted rape
where there wasno penetrationof the female organ because not all acts of execution were
performed as the offender merely commenced the commission of a felony directly by overt acts.
The inference that may be derived therefrom is that complete or full penetration of the vagina is
not required for rape to be consummated. Any penetration, in whatever degree, is enough to raise
the crime to its consummated stage.
But the Court inOritaclarified the concept of penetration in rape by requiring entry into
thelabiaor lips of the female organ, even if there be no rupture of the hymen or laceration of the
vagina, to warrant a conviction for consummated rape. While the entry of the penis into the lips
of the female organ was considered synonymous withmere touchingof the external genitalia,
e.g.,labia majora,labia minora, etc., the crucial doctrinal bottom line is thattouchingmust be
inextricably viewed in light of, in relation to, or as an essential part of, the process of penile
penetration, andnot just mere touching in the ordinary sense. In other words, thetouchingmust
be tacked to the penetration itself. The importance of the requirement of penetration, however
slight, cannot be gainsaid because where entry into thelabiaor the lips of the female genitalia
has not been established, the crime committed amounts merely to attempted rape.
Where the accused failed to achieve an erection, had a limp or flaccid penis, or an oversized penis
which could not fit into the victims vagina, the Court nonetheless held that rape was
consummated on the basis of the victims testimony that the accused repeatedly tried, but in
vain, to insert his penis into her vagina and in all likelihood reached thelabiaof herpudendumas
the victim felt his organ on the lips of her vulva, or that the penis of the accused touched the
middle part of her vagina. Thus,touchingwhen applied to rape cases does not simply mean mere
epidermal contact, stroking or grazing of organs, a slight brush or a scrape of the penis on the
external layer of the victims vagina, or themons pubis, as in this case. There must be sufficient
and convincing proof that the penis indeedtouchedthelabiasorslidinto the female organ,
andnot merely stroked the external surface thereof, for an accused to be convicted of
consummated rape. As thelabias, which are required to be touched by the penis, are by their
naturalsitusor location beneath themons pubisor the vaginal surface, to touch them with the
penis is to attain some degree of penetration beneath the surface, hence, the conclusion that
touching thelabia majoraor thelabia minoraof thepudendumconstitutes consummated rape.
Thus, a grazing of the surface of the female organ or touching themons pubisof thepudendumis
not sufficient to constitute consummated rape. Absent any showing of the slightest penetration of
the female organ, i.e., touching of eitherlabiaof thepudendumby the penis, there can be no
consummated rape; at most, it can only be attempted rape, if not acts of lasciviousness.

G.R. No. 129433 March 30, 2000

Lessons Applicable: Attempted rape

Laws Applicable:

FACTS:
April 25, 1996 4 pm: Ma. Corazon P. Pamintuan, mother of 4-year old Crysthel Pamintuan, went
to the ground floor of their house to prepare Milo chocolate drinks for her 2 children. There she met
Primo Campuhan, helper of Conrado Plata Jr., brother of Corazon, who was then busy filling small
plastic bags with water to be frozen into ice in the freezer located at the second floor.
Then she heard Crysthel cry, "Ayo'ko, ayo'ko!" so she went upstairs and saw Primo Campuhan
inside her children's room kneeling before Crysthel whose pajamas or "jogging pants" and panty
were already removed, while his short pants were down to his knees and his hands holding his penis
with his right hand
Horrified, she cursed "P - t - ng ina mo, anak ko iyan!" and boxed him several times. He evaded
her blows and pulled up his pants. He pushed Corazon aside who she tried to block his path.
Corazon then ran out and shouted for help thus prompting Vicente, her brother, a cousin and an
uncle who were living within their compound, to chase the Campuhan who was apprehended. They
called the barangay officials who detained.
Physical examination yielded negative results as Crysthel s hymen was intact
Campuhan: Crysthel was in a playing mood and wanted to ride on his back when she suddenly
pulled him down causing both of them to fall down on the floor.
RTC: guilty of statutory rape, sentenced him to the extreme penalty of death
Thus, subject to automatic review

ISSUE: W/N it was a consummated statutory rape

HELD: NO. MODIFIED. guilty of ATTEMPTED RAPE and sentenced to an indeterminate prison term
of eight (8) years four (4) months and ten (10) days of prision mayor medium as minimum, to
fourteen (14) years ten (10) months and twenty (20) days of reclusion temporal medium as
maximum. Costs de oficio.

People v. De la Pea: labia majora must be entered for rape to be consummated


Primo's kneeling position rendered an unbridled observation impossible
Crysthel made a categorical statement denying penetration but her vocabulary is yet as
underdeveloped
Corazon narrated that Primo had to hold his penis with his right hand, thus showing that he had
yet to attain an erection to be able to penetrate his victim
the possibility of Primo's penis having breached Crysthel's vagina is belied by the child's own
assertion that she resisted Primo's advances by putting her legs close together and that she did not
feel any intense pain but just felt "not happy" about what Primo did to her. Thus, she only shouted
"Ayo'ko, ayo'ko!" not "Aray ko, aray ko!
no medical basis to hold that there was sexual contact between the accused and the victim
FACTS:

February 5, 1988 11:30 pm: Elisa Rolan was inside their store waiting for her husband to
arrive. Joselito Capa and Julian Azul, Jr. were drinking beer. Although already drunk, Edmar
Aguilos and Odilon Lagliba joined them. Edmar had a heated argument with Julian. Elisa
pacified Edmar and advised them to go home as she was already going to close up. Edmar
and Odilon left then returned to block Joselito and Julian. Edmar took off his eyeglasses and
punched Julian in the face. Elisa shouted: Tama na. Tama na but she was ignored as they
continued until they reached the end of the street. Odilon positioned himself on top of a pile
of hollow blocks and watched as Edmar and Julian swapped punches. As Joselito tried to
stop the fight, Odilon pulled out his knife with his right hand and stepped down from his
perch. He placed his left arm around Joselitos neck, and stabbed him. Ronnie and Rene
Gayot Pilola, who were across the street, saw their gangmate Odilon stabbing the victim and
decided to join the fray. Ronnie took a knife from the kitchen of Teresita and rushed together
with Pilola to the scene and stabbed Joselito. As Joeslito was stabbed 11 times (6 fatal stab
wounds), he fell in the canal. Odilon and Pilola fled while Ronnie went after Julian who ran
dear life. When Julian noticed that Ronnie was no longer running after him, he looked back
and saw Ronnie pick up a piece of hollow block and bashed Joselitos head. Then, Ronnie
got a piece of broken bottle and struck Joselito once more before fleing from the
scene. Joselito died on the spot. Elisa rushed to Joselitos house and informed his wife and
brother of the incident.
Agripina Gloria, a female security guard, saw Ronnie repeatedly stabbed Joselito and fled
towards the direction of the mental hospital. She did not see Odilon.
Elisa cross-examination had an inconsistency, she stated that it was Edmar who struck the
victim (before it was Ronnie)
RTC: Pilola GUILTY beyond reasonable doubt of Murder qualified by treachery and
sentenced to reclusion perpetua

ISSUE: W/N Pilola is guilty of murder

HELD: YES. Rene Gayot Pilola GUILTY beyond reasonable doubt of the crime of murder is
AFFIRMED WITH MODIFICATION

The identity of the person who hit the victim with a hollow block is of de minimis
importance. Elisas testimony is corroborated by the autopsy report of Dr. Bienvenido
Muoz. No showing of any improper motive on the part of a witness to testify falsely against
the accused or to falsely implicate the latter in the commission of the crime. The trial court
gave credence and full probative weight to Elisas testimony.
There is conspiracy when two or more persons agree to commit a felony and decide to
commit it. Conspiracy as a mode of incurring criminal liability must be proved separately
from and with the same quantum of proof as the crime itself. Conspiracy need not be proven
by direct evidence. After all, secrecy and concealment are essential features of a successful
conspiracy. It may be inferred from the conduct of the accused before, during and after the
commission of the crime, showing that they had acted with a common purpose and design.
Conspiracy may be implied if it is proved that two or more persons aimed by their acts
towards the accomplishment of the same unlawful object, each doing a part so that their
combined acts, though apparently independent of each other, were, in fact, connected and
cooperative, indicating a closeness of personal association and a concurrence of sentiment.
There may be conspiracy even if an offender does not know the identities of the other
offenders, and even though he is not aware of all the details of the plan of operation or was
not in on the scheme from the beginning. One need only to knowingly contribute his efforts in
furtherance of it. One who joins a criminal conspiracy in effect adopts as his own the criminal
designs of his co-conspirators. If conspiracy is established, all the conspirators are liable as
co-principals regardless of the manner and extent of their participation since in contemplation
of law, the act of one would be the act of all. Each of the conspirators is the agent of all the
others.
The mere presence of an accused at the situs of the crime will not suffice. There must be
intentional participation in the transaction with a view to the furtherance of the common
design and purpose.
Even if two or more offenders do not conspire to commit homicide or murder, they may be
held criminally liable as principals by direct participation if they perform overt acts which
mediately or immediately cause or accelerate the death of the victim. Art. 4. Criminal
liability. Criminal liability shall be incurred:
o1. By any person committing a felony (delito) although the wrongful act done be different from that
which he intended.

Art. 18. Accomplices. Accomplices are the persons who, not being included in Article 17,
cooperate in the execution of the offense by previous or simultaneous acts.
oTo hold a person liable as an accomplice, two elements must concur:

1.the community of criminal design; that is, knowing the criminal design of the principal by direct
participation, he concurs with the latter in his purpose;

2.the performance of previous or simultaneous acts that are not indispensable to the commission
of the crime

Accomplices come to know about the criminal resolution of the principal by direct
participation after the principal has reached the decision to commit the felony and only then
does the accomplice agree to cooperate in its execution. Accomplices do not decide
whether the crime should be committed; they merely assent to the plan of the principal by
direct participation and cooperate in its accomplishment
However, where one cooperates in the commission of the crime by performing overt acts
which by themselves are acts of execution, he is a principal by direct participation, and not
merely an accomplice
Odilon all by himself initially decided to stab the victim. However, while Odilon was stabbing
the victim, the appellant and Ronnie agreed to join. All the overt acts of Odilon, Ronnie and
the Pilola before, during, and after the stabbing incident indubitably show that they conspired
to kill the victim. Since the victim is not yet dead, the crime is not yet consummated so Pilola
is a principal by direct participation.
Alibi is a weak, if not the weakest of defenses in a criminal prosecution, because it is easy to
concoct but hard to disprove. To serve as basis for acquittal, it must be established by clear
and convincing evidence. For it to prosper, the accused must prove not only that he was
absent from the scene of the crime at the time of its commission, but also that it was
physically impossible for him to have been present then.
Pilola knew that he was charged for the stabbing but instead of surrendering to the police
authorities, he evaded arrest and this flight is evidence of guilt
There is treachery when the offender commits any of the crimes against persons, employing
means, methods or forms in the execution thereof which tend directly and specially to insure
its execution, without risk to himself arising from the defense which the offended party might
make. The essence of treachery is the swift and unexpected attack on the unarmed victim
without the slightest provocation on his part - attack on the unarmed victim was
sudden. The aggravating circumstance of abuse of superior strength is absorbed by
treachery
PEOPLE v. DECENA
BY MAROON 5 PARTNERS AND ASSOCIATESJUNE 1, 2012ART. 11 RPC SELF-DEFENSE

Facts:

On Christmas day, around 4pm, Luzviminda (14 y.o., daughter of the Jaime
Ballesteros, victim), saw Decena rushing towards her father with a long bladed
weapon prompting her to warn her father.
Decena, however, stabbed him on the right chest causing his death.
Narration of the defense:
At about 4pm, the victim(Jaime) was drunk and for no apparent reason, he
held the appellant by the neck and poked a fork against it. A barangay tanod
intervened and advised the appellant to go home. Appellant left but was
later followed by Jaime (victim).
Biala, uncle of the appellant, testified that he saw Jaime attacking the
appellant with a balisong. Appellant was able to parry the blow, and
overpowering Jaime, thruste the knife into his body.
Issue: W/N the appellant acted in complete self-defense that in killing Jaime
Ballesteros absolving him from criminal liability.

Held: No.

Ratio:

In criminal cases, the burden of proof is on the prosecution which may rely on
the strength of its evidence and not on the weakness of the defense. However,
upon invoking self-defense, the accused admits that he killed the victim and the
burden of proof is upon him in proving that he really acted in self-defense.
Basic requirement for self-defense as a justifying circumstance is unlawful
aggression against the person defending himself.
It must be shown that there was a previous unlawful and unprovoked attack
that placed the life of the accused in danger forcing him to inflict wounds
upon his assailant
According to the defense, the unlawful aggression started when the victim
started poking the appellant with a fork
Elementary rule: when the aggressor leaves, the aggression ceases. It
follows that when appellant and Jaime heeded the advice of the
barangay tanod, the unlawful aggression had ended. Since the
aggression no longer existed, appellant had no right to kill or even
wound the former aggressor.
The defense failed to establish that the victim persisted in his design to
attack the appellant
Defense: continuing aggression
Whenever the victim was drunk, he would look for trouble (refuted by the
testimony of the wife)
Witnesses: Jaime was staggering or wobbling as he walked the victim
could not have persisted in attacking the appellant with his current state.
Testimony of the uncle: imaginative or coached witness
People vs. Narvaez, 121 SCRA 389 (1983)
FACTS: Mamerto Narvaez has been convicted of murder (qualified by treachery) of David Fleischer and
Flaviano Rubia. On August 22, 1968, Narvaez shot Fleischer and Rubia during
the time the two were constructing a fence that would prevent Narvaez from getting into his house and
rice mill. The defendant was taking a nap when he heard sounds of construction and
found fence being made. He addressed the group and asked them to stop destroying his house and
asking if they could talk things over. Fleischer responded with "No, gadamit, proceed, go
ahead." Defendant lost his "equilibrium," and shot Fleisher with his shotgun. He also shot Rubia who was
running towards the jeep where the deceased's gun was placed. Prior to the
shooting, Fleischer and Co. (the company of Fleischer's family) was involved in a legal battle with the
defendant and other land settlers of Cotabato over certain pieces of property. At the time
of the shooting, the civil case was still pending for annulment (settlers wanted granting of property to
Fleisher and Co. to be annulled). At time of the shooting, defendant had leased his
property from Fleisher (though case pending and ownership uncertain) to avoid trouble. On June 25,
defendant received letter terminating contract because he allegedly didn't pay rent.
He was given 6 months to remove his house from the land. Shooting was barely 2 months after letter.
Defendant claims he killed in defense of his person and property. CFI ruled that
Narvaez was guilty. Aggravating circumstances of evident premeditation offset by the mitigating
circumstance of voluntary surrender. For both murders, CFI sentenced him to reclusion perpetua, to
indemnify the heirs, and to pay for moral damages.

ISSUES:
1. Whether or not CFI erred in convicting defendant-appellant despite the fact that he acted in defense of
his person.

No. The courts concurred that the fencing and chiselling of the walls of the house of the defendant was
indeed a form of aggression on the part of the victim. However, this
aggression was not done on the person of the victim but rather on his rights to property. On the first issue,
the courts did not err. However, in consideration of the violation of property rights, the courts referred to
Art. 30 of the civil code recognizing the right of owners to close and fence their land.

Although is not in dispute, the victim was not in the position to subscribe to the article because his
ownership of the land being awarded by the government was still pending, therefore putting ownership
into question. It is accepted that the victim was the original aggressor.

2. WON the court erred in convicting defendant-appellant although he acted in defence of his rights.

Yes. However, the argument of the justifying circumstance of self-defense is applicable only if the 3
requirements are fulfilled. Art. 11(1) RPC enumerates these requisites:
Unlawful aggression. In the case at bar, there was unlawful aggression towards appellant's
property rights. Fleisher had given Narvaez 6 months and he should have left him in peace before
time was up, instead of chiseling Narvaez's house and putting up fence. Art. 536 of the Civil Code
also provides that possession may not be acquired through force or intimidation; while Art. 539
provides that every possessor has the right to be respected in his possession
Reasonable necessity of means employed to prevent or repel attack. In the case, killing was
disproportionate to the attack.
Lack of sufficient provocation on part of person defending himself. Here, there was no
provocation at all since he was asleep
Since not all requisites present, defendant is credited with the special mitigating circumstance of
incomplete defense, pursuant to Art. 13(6) RPC. These mitigating circumstances are: voluntary surrender
and passion and obfuscation (read p. 405 explanation) Crime is homicide (2 counts) not murder because
treachery is not applicable on account of provocation by the deceased. Also, assault was not deliberately
chosen with view to kill since slayer acted instantaneously. There was also no direct evidence of planning
or preparation to kill. Art. 249 RPC: Penalty for homicide is reclusion temporal. However, due to mitigating
circumstances and incomplete defense, it can be lowered three degrees (Art. 64) to arrestomayor.

3. WON he should be liable for subsidiary imprisonment since he is unable to pay the civil indemnity due
to the offended party.

No. He is not liable to be subsidiarily imprisoned for nonpayment of civil indemnity. RA 5465 made the
provisions of Art. 39 applicable to fines only and not to reparation of damage caused, indemnification of
consequential damages and costs of proceedings. Although it was enacted only after its conviction,
considering that RA 5465 is favorable to the accused who is not a habitual delinquent, it may be given
retroactive effect pursuant to Art. 22 of the RPC.

Judgment: Defendant guilty of homicide but w/ mitigating circumstances and extenuating circumstance of
incomplete self defense. Penalty is 4 months arresto mayor and to indemnify
each group of heirs 4,000 w/o subsidiary imprisonment and w/o award for moral damages. Appellant has
already been detained 14 years so his immediate release is ordered.

Gutierrez, dissenting. Defense of property can only be invoked when coupled with form of attack on
person defending property. In the case at bar, this was not so. Appellant should then be sentenced to
prision mayor. However, since he has served more than that, he should be released.
Crim Law 1 Case Digest: People V. Palaganas
2006

FACTS:
January 16, 1998 8pm: Brothers Servillano, Melton and Michael Ferrer were on a drinking spree
in their house because Melton visited his brothers in Pangasinan all the way from San Fernando, La
Union.
January 16, 1998 9:45 pm: The brothers decided to go to Tidbits Videoke bar to continue their
drinking spree and to sing. They were the only customers
January 16, 1998 10:30 pm: Jaime Palaganas, Ferdinand Palaganas and Virgilio Bautista arrived
and they occupied a different table. When Jaime sang My Way, Melton sang along. But, Jaime
resented this, approached the brother and said in Pangasinan dialect "As if you are tough guys. You
are already insulting me in that way." Jaime struck Servillanos head with the microphone and a fight
ensued. Virgilio Bautista did not joined in and just left. During the rumble, Ferdinand went out of the
bar. Michael was about to pursue him but was stopped by Servillano. They went back to continue to
fight with Jaime. Edith Palaganas, sister of Jaime and the owner of the bar, arrived and pacified
them. Servillano noticed that his wristwatch was missing. Since the brothers could not locate it
inside the bar, they went outside. They saw Ferdinand at them and said to Rujjeric Palaganas
"Oraratan paltog mo lara" meaning "They are the ones, shoot them." Rujjeric shot Servillano first at
the left side of the abdomen penetrating his large intestine and urinary bladder causing him to fall on
the ground then Melton with a fatal shot on the head and on the right thigh. When Servillano noticed
that Melton was no longer moving, he told Michael "Bato, bato and they threw stones at Rujjeric and
Ferdinand. Michael was hit on the right shoulder.
The police came and took the Ferrer brothers to Manaoag Hospital and later to Villaflor Hospital
in Dagupan.
Criminal Case No. U-9608: Shooting Servillano with unlicensed firearm
Criminal Case No. U-9609: Shooting Melton with unlicensed firearm
Criminal Case No. U-9610: Shooting Michael with unlicensed firearm
Criminal Case No. U-9634: using a caliber .38 without first securing the necessary permit/license
in violation to Comelec Res. 2958
Rujjeric and Ferdinand entered separate pleas of "Not Guilty" Upon motion of Ferdinand, the four
cases were consolidated.
RTC: Rujjeric was guilty of the crime of Homicide and 2 counts of Frustrated Homicide but
acquitted of the charge of Violation of COMELEC Resolution No. 2958 in relation to Section 261 of
the Omnibus Election Code while Ferdinand was acquitted of all the charges against him.
CA Affirmed
Rujjeric argued that all the elements of a valid self-defense are present in the instant case and,
thus, his acquittal on all the charges is proper; that when he fired his gun, he was then a victim of an
unlawful aggression perpetrated by the Ferrer brothers; that he, in fact, sustained an injury in his left
leg and left shoulder caused by the stones thrown by the Ferrer brothers

ISSUE: W/N Rujjeric was guilty of the crime of Homicide and 2 counts of Frustrated Homicide

HELD: YES. AFFIRMED with the following MODIFICATIONS:


Criminal Case No. U-9608: Shooting Servillano with unlicensed firearm - attempted homicide.
There being a special aggravating circumstance of the use of an unlicensed firearm and applying the
Indeterminate Sentence of Law, the penalty now becomes four (4) years and two (2) months of
arresto mayor as minimum period to six (6) years of prision correccional as maximum period
Criminal Case No. U-9609: Shooting Melton with unlicensed firearm - homicide is reclusion
temporal - There being a special aggravating circumstance of the use of an unlicensed firearm and
applying the Indeterminate Sentence Law, the penalty now is twelve (12) years of prision mayor as
minimum period to twenty (20) years of reclusion temporal as maximum period
Criminal Case No. U-9610: Shooting Michael with unlicensed firearm - frustrated homicide. There
being a special aggravating circumstance of the use of an unlicensed firearm and applying the
Indeterminate Sentence Law, the penalty now becomes six (6) years of prision correccional as
minimum period to twelve (12) years of prision mayor as maximum period.

petitioner argued that all the elements of a valid self-defense are present in the instant case and,
thus, his acquittal on all the charges is proper; that when he fired his gun on that fateful night, he
was then a victim of an unlawful aggression perpetrated by the Ferrer brothers; that he, in fact,
sustained an injury in his left leg and left shoulder caused by the stones thrown by the Ferrer
brothers
ART. 11. Justifying circumstances. The following do not incur any criminal liability:
1. Anyone who acts in defense of his person or rights, provided that the following circumstances
concur;
First. Unlawful aggression;
o no unlawful aggression on the part of the Ferrer brothers that justified the act of petitioner in
shooting them. Ferrer brothers then were merely standing outside the videoke bar and were not
carrying any weapon
o When the Ferrer brothers started throwing stones, petitioner was not in a state of actual or
imminent danger considering the wide distance (4-5 meters) of the latter from the location of the
former. He was still capable of avoiding the stones by running away or by taking cover. He could
have also called or proceeded to the proper authorities for help
Second. Reasonable necessity of the means employed to prevent or repel it;
o gun was far deadlier compared to the stones thrown by the Ferrer brothers.
Third. Lack of sufficient provocation on the part of the person defending himself. x x x.
unlawful aggression is a primordial element in self-defense. It is an essential and indispensable
requisite, for without unlawful aggression on the part of the victim
As the burden of evidence is shifted on the accused to prove all the elements of self-defense, he
must rely on the strength of his own evidence and not on the weakness of the prosecution
1.) In frustrated felony, the offender has performed all the acts of execution which should produce
the felony as a consequence; whereas in attempted felony, the offender merely commences the
commission of a felony directly by overt acts and does not perform all the acts of execution.
2.) In frustrated felony, the reason for the non-accomplishment of the crime is some cause
independent of the will of the perpetrator; on the other hand, in attempted felony, the reason for the
non-fulfillment of the crime is a cause or accident other than the offender's own spontaneous
desistance.
when the accused intended to kill his victim, as manifested by his use of a deadly weapon in his
assault, and his victim sustained fatal or mortal wound/s but did not die because of timely medical
assistance, the crime committed is frustrated murder or frustrated homicide depending on whether or
not any of the qualifying circumstances under Article 249 of the Revised Penal Code are present.
However, if the wound/s sustained by the victim in such a case were not fatal or mortal, then the
crime committed is only attempted murder or attempted homicide.
If there was no intent to kill on the part of the accused and the wound/s sustained by the victim
were not fatal, the crime committed may be serious, less serious or slight physical injury
Michals wound took six to eight days to heal - attempted homicide
use of an unlicensed firearm - special aggravating circumstance by Republic Act. No. 8294 on
June 6, 1997
Generic aggravating circumstances are those that generally apply to all crimes such as those
mentioned in Article 14, paragraphs No. 1, 2, 3, 4, 5, 6, 9, 10, 14, 18, 19 and 20, of the Revised
Penal Code. It has the effect of increasing the penalty for the crime to its maximum period, but it
cannot increase the same to the next higher degree. It must always be alleged and charged in the
information, and must be proven during the trial in order to be appreciated. Moreover, it can be offset
by an ordinary mitigating circumstance. On the other hand, special aggravating circumstance,
CANNOT be offset by an ordinary mitigating circumstance
PEOPLE V RICOHERMOSO
11 FEB

L 30527 28| March 29, 1974 | J. Aquino


Avoidance of Greater Evil or Injury
Facts:
Geminiano de Leon, together with his common-law wife, son Marianito de Leon and one Rizal
Rosales, chanced upon Pio Ricohermoso. Owning a parcel of land, which Ricohermoso cultivated as
kaingin, Geminiano asked about his share of palay harvest and added that she should be allowed to
taste the palay harvested from his land. Ricohermoso said Geminiano could collect the palay
anytime.

Upon returning from his trip to Barrio Bagobasin, Geminiano dropped by Ricohermosos house and
asked him about the palay, to which the latter answered defiantly that he will not give him the palay,
whatever happens. Geminiano remonstrated and that point (as if by prearrangement), Ricohermoso
unsheathed his bolo, while his father-in-law Severo Padernal got an axe, and attacked Geminiano. At
the same time and place, Ricohermosos brother-in-law Juan Padernal suddenly embraced
Marianito. They grappled and rolled down the hill, at which point Marianito passed out. When he
regained consciousness, he discovered that the rifle he carried beforehand was gone and that his
father was mortally wounded.

The defendants shifted the responsibility of killing in their version of the case.

Issue:
W/N appellant Juan Padernal can invoke the justifying circumstance of avoidance of a greater evil or
injury

Held:
No.Juan Padernals reliance on the justifying circumstance is erroneous because his act in
preventing Marianito from shooting Ricohermoso and Severo Padernal, the aggressors in this case,
was designed to insure the killing of Geminiano de Leon without any risk to the assailants andnot an
act to prevent infliction of greater evil or injury. His intention was to forestall any interference in the
assault.
Treachery was also appreciated in the case. The trial court convicted the appellants withlesiones
leves, from an attempted murder charge with respect to Marianito de Leon.
Judgment as to Juan Padernal affirmed.

(Note: Severo Padernal withdrew his appeal, thus, in effect, accepted the prosecutions version of the
case and trial courts finding of guilt.)
PEOPLE VS. LAGATA (1949)
BY MAROON 5 PARTNERS AND ASSOCIATESJUNE 1, 2012ESCAPING PRISONERS

Plaintiff-Appellee: People of the Philippines

Defendant-Appellant: Ignacio Lagata

FACTS:

The accused, Ignacio Lagata, a provincial guard of Catbalogan, Samar, was in


charge of 6 prisoners (Jesus, Tipace, Eusebio, Mariano, Labong & Abria)
assigned to work in the capitol plaza of Samar.
Lagata ordered the prisoners to go to the nursery to pick up gabi. Not long
afterwards, they were called to assemble. Epifanio Labong was missing so
Lagata ordered the 5 remaining prisoners to go look for him.
Eusebio Abria said that while they were gathering gabi, he heard 3 shots. He
was wounded by the 2nd one. They were already assembled by the 1st shot and
that he did not see Tipace being shot. He said he ran away because he was
afraid that he might be shot again and that his companions were also probably
scared and that is why they ran.
Another prisoner, Mariano Ibaez stated that Epifanio Labong did not answer
their call so Ignacio Lagata ordered to go look for him in the mountain. He
said that Abria went to the camote plantation and found footprints and called on
Lagata to inform him about the footprints. When Abria told Lagata of the
flattened grass and that he was unable to look for Labong, Ignacio Lagata fired
at him and he was hit on his left arm. Abria told Lagata he was wounded and
in turn, Lagata told them to assemble. Once they were assembled, Lagata
cocked his gun and shot Ceferino Tipace. Mariano said that when he saw
Tipace was shot, he ran away because he also could have been shot.
Eustaquio Galet, another detainee, received good treatment from Lagata though
his testimony corroborated those of the other prisoners.
Pedro Mayuga, chief of Samar Provincial Hospital & Gilberto Rosales,
Sanitary Division president, verified the gunshot wound and that the death of
Tipace resulted therein.
Ignacio Lagata, however, said that he fired his gun because the prisoners were
running far from him when he already ordered them to stop. He said that he
would be the one in jail if a prisoner escaped under his custody. Furthermore,
he would be discharged from duty like the others. He was hopeless already.
Moreover, the picking up of gabi was not part of the prisoners work.
HELD:

Court ruled that Lagata should be sentenced for homicide and serious physical
injuries.
Appellant was entitled to the benefit of mitigating circumstance of incomplete
justifying circumstance. (Art.11par.5, RPC)
RATIO:

It was clear that Lagata had absolutely no reason to fire at Tipace. The record
does not show that Tipace was bent on committing any act of aggression or that
he attempted to escape.
According to Lagata himself, Tipace was running towards and around him.
How could anyone intending to escape run towards and around the very guard
one was supposed to escape from?
Even if Lagata sincerely believed that he acted in the performance of his duties,
the circumstances show that there was no necessity for him to fire directly
against the prisoners as to wound them seriously and even kill one of them.
While custodians should take care for prisoners not to escape, only
ABSOLUTE NECESSITY would authorize them to fire against them.
ALP

MAMANGUN V PEOPLE
11 FEB

GR No. 149152| February 2, 2007 | J. Garcia


Fulfillment of Duty/Lawful Exercise of Right
Facts:
Policeman (PO2) Rufino Mamangun was responding to a robbery-holdup call, with his fellow police
officers, at Brgy. Calvario, Meycauayan, Bulacan. A certain Liberty Contreras was heard shouting,
which prompted residents to respond and chase the suspect, who entered the yard and proceeded to
the rooftop of Antonio Abacan. Mamangun, with PO2 Diaz and Cruz, each armed with a drawn
handgun, searched the rooftop and saw a man who they thought was the robbery suspect.
Mamangun, who was ahead of the group, fired his gun once and hit the man, who turned out to be
Gener Contreras (not the suspect) Contreras died of the gunshot wound.

According to the lone witness Crisanto Ayson, he accompanied the policemen to the lighted rooftop.
He was beside Mamangun when he (Ayson) recognized the deceased. According to Ayson,
Mamangun pointed his gun at the man, who instantly exclaimed Hindi ako, hindi ako! to which
Mamangun replied, Anong hindi ako? and shot him.

The defense rejects this testimony, alleging that they were the only ones at the dark rooftop when
Mamangun noticed a crouching man who suddenly continued to run. Mamangun shouted Pulis,
tigil! whereupon the person stopped and raised a steel pipe towards Mamanguns head. This
prompted Mamangun to shoot the person. The three police claim that Contreras only said Hindi
ako, hindi ako only when they approached him. Mamangun then asked Why did you go to the
rooftop? You know there are policemen here. Mamangun reported the incident to the desk officer
who directed investigator Hernando Banez to investigate the incident. Banez later on found a steel
pipe on the roof.

Issue:
W/N the death of the victim was the necessary consequence of the petitioners fulfillment of his duty

Held:
No. The Court denies the instant petition and affirms Sandiganbayans decision after finding the
petitioners testimony to be nothing but a concocted story designed to evade criminal liability. Per
Sandiganbayans observations, the defense was self-serving for the accused and biased with respect
to his co-policemen-witnesses because:
1. After supposed introductions and forewarnings uttered allegedly by Mamangun, it is contrary to
human experience for a man (who is not the suspect) to attack one of three policemen with
drawn guns
2. Mamanguns admission that he did not ask the victim Why did you try to hit me, if you are not
the one? clearly belies their claim
3. The location of the entry of bullet belies their claim because it appears that the victim
instinctively shielded himself instead
Additionally, petitioners pretense that Contreras struck him was not initially reported to the desk
and was only conveniently remembered when the investigator found a pipe in the crime scene.

Acts in the fulfillment of duty and self-defense does not completely justify the petitioners firing the
fatal gunshot. The element of unlawful aggression on the part of the victim was absent, which leads
to the failure of the petitioners plea. Also, there can only be incomplete justification (a privileged
mitigating circumstance) in the absence of a necessary justifying circumstance the injury was caused
by necessary consequence of due performance of duty.
"
People v. Beronilla, et al.
BY MAROON 5 PARTNERS AND ASSOCIATESJUNE 1, 2012ART. 11 RPC

Plaintiff-appellee: People of the Phil.

Defedants-appellants: Manuel Beronilla, Filipino Velasco, Policarpo Paculdo,


Jacinto Adriatico

Facts:

Setting: After Japanese occupation


Borjal served as mayor during the Japanese occupation.
Beronilla, after being appointed as Military Mayor of La Paz, Abra, received a
copy of a memorandum issued by Lt. Col. Arnold authorizing all military
mayors to appoint a hury of 12 bolomen to try persons accused of treason,
espionage or the aiding of the enemy.
He also received a list of all puppet officials of the province of Abra (Borjal
included) with a memorandum instructing all Military Mayors to investigate
said persons and gather against them complaints from people of the
municipality.
Beronilla, upon the return of Borjal who left La Paz because of an attempt on
his life, placed Borjal under custody.
Pursuant to his instructions, complaints were gathered, a 12-man jury was
appointed, prosecutors and a clerk of the jury were assigned.
Trial lasted for 19 days and the jury found Borjal guilty on all counts
(espionage, aiding the enemy, abuse of authority). Death penalty was imposed.
Beronilla forwarded the records of the case to the Headquarters of the 15th
Infantry for review.
The records were returned by Lt. Col. Arnold adding that the matter was best
handled by the La Paz Government and whatever disposition taken was
approved.
Upon receipt of the letter, Beronilla then ordered the execution of Borjal. The
execution was reported to Col. Arnold and Beronilla received compliments
based on the reply of his superior.
Two years after, those who were involved were indicted in the CFI of Abra for
murder for allegedly conspiring and confederating in the execution of Borjal.
The defendants were convicted, thus this appeal.
Issue: Are the defendants guilty of murdering Borjal?

Held: No.

Ratio Decidendi:

The records are ample to sustain the claim of the defense that the arrest,
prosecution and trial of Borjal were done pursuant to express orders of the 15th
Infantry HQ.
The state however contends that there was a radiogram from Col. Volckmann
to Lt. Col. Arnold, specifically noting the illegality of Borjals conviction and
sentence, which the prosecution claims that Beronilla was informed about this
but still pursued with the execution.
A witness, a relative of Borjal, asserts that he was present when Beronilla
received the message and was able to read it over Beronillas shoulder. But
basing on his affidavit, it can be imported that witness was not with Beronilla
the message alleged to have arrived.
Moreover, Beronilla would not have informed Lt. Col. Arnorld about the
execution if he did it after the receipt of the message since this would be in
violation of superior orders. And more importantly, Arnold complimented him
in his reply.
The Court concludes that Lt. Col. Arnold failed to transmit the Volckmann
message to Beronilla. The charge of criminal conspiracy to do away with
Borjal must be rejected because the accused had no need to conspire against a
man who was, to their knowledge, duly sentenced to death.
The accused acted as military subordinates only following superior orders, thus
there is no criminal intent.
Judgement appealed from reversed and the appellants were acquitted.

PEOPLE V BERONILLA
11 FEB

L 4445| February 28, 1955 | J. JBL Reyes


Obedience to Lawful Order of a Superior
Facts:
Manuel Beronilla, Policarpio Paculdo, Filipino Velasco and Jacinto Adriatico file an appeal from the
judgement of the Abra CFI, which convicted them of murder for the execution of Arsenio Borjal, the
elected mayor of La, Paz, Abra (at the outbreak of war), which was found to be aiding the enemy.
Borjal moved to Bangued because of death threats was succeeded by Military Mayor Manuel
Beronilla, who was appointed by Lt. Col. Arbold, regimental commander of the 15thInfantry of the
Phil. Army, operating as guerilla unit in Abra. Simultaneously upon his appointment, Beronilla
received a memorandum which authorized him to appoint a jury of 12 bolo men to try persons
accused of treason, espionage and aiding or abetting the enemy.
Upon the return of Borjal and his family to Abra, to escape bombing in Bangued, he was placed
under custody and tried and sentenced to death by the jury based on various complaints made by the
residents. Beronilla reported this to Col. Arnold who replied, saying I can only compliment you for
your impartial but independent way of handling the whole case.

Two years thereafter, Beronilla, along with the executioner, digger and jury, were indicted for the
murder of Borjal. Soon after, President Manuel Roxas issued Executive Proclamation 8, which
granted amnesty to persons who committed acts in furtherance of the resistance to the enemy
against persons aiding in the war efforts of the enemy.

The rest of defendants applied and were granted amnesty, but Beronilla and others were convicted
on the grounds that the crime was made on purely personal motives and that the crime was
committed after the expiration of time limit for amnesty proclamation.

Issue: W/N the defendant-appellants actions are covered by justifying circumstances for obedience
to lawful order of superior

Held:
Yes. The accused acted upon orders of their superior officers, which as military subordinates, they
could not question and obeyed in good faith without the being aware of its illegality.
The evidence is sufficient to sustain the claim of the defense that arrest, prosecution and trial of
Borjal was done in pursuant to express orders of superiors. Additionally, it could not be established
that Beronilla received the radiogram from Colonel Volckmann, overall area commander, which
called attention to the illegality of Borjals conviction and sentence. Had Beronilla known the
violation, he would not have dared to report it to Arnold. The conduct of the accused also does not
show malice on their part because of the conduct of the trial, defense through counsel given to Borjal,
suspension of trial based on doubts of illegality and death sentence review sent to the superior
officers.
Criminal intent then could not be established. The maxim here isactus non facit reum, nisi mens
rea (Crime is not committed if the mind of the person performing the act complained of to be
innocent).
Additionally, the lower court should not have denied their claim to the benefits of the Guerilla
Amnesty Proclamation No. 8 inspite of contradictory dates of liberation of La Paz, Abra. Even if the
dates were contradictory, the court should have found for the Beronila, et al because if there are any
reasonable doubt as to whether a given case falls within the (amnesty) proclamation should be
resolved in favor of the accused.

Judgement reversed, appellants acquitted.


Advertisements

Case Brief: Tabuena vs. Sandiganbayan


MAY 15, 2017JEFF REY

G.R. Nos. 103501-03 February 17, 1997

LUIS A. TABUENA, petitioner,


vs.
HONORABLE SANDIGANBAYAN, and THE PEOPLE OF THE
PHILIPPINES, respondents.

G.R. No. 103507 February 17, 1997

ADOLFO M. PERALTA, petitioner,


vs.
HON. SANDIGANBAYAN (First Division), and THE PEOPLE OF THE
PHILIPPINES, represented by the OFFICE OF THE SPECIAL
PROSECUTOR, respondents.

Facts:
Then President Marcos instructed Luis Tabuena over the phone to pay directly to the
presidents office and in cash what the Manila International Airport Authority (MIAA)
owes the Philippine National Construction Corporation (PNCC), pursuant to the 7
January 1985 memorandum of then Minister Trade and Industry Roberto Ongpin.
Tabuena agreed. About a week later, Tabuena received from Mrs. Fe Roa-Gimenez,
then private secretary of Marcos, a Presidential Memorandum dated 8 January 1986
reiterating in black and white such verbal instruction. In obedience to President
Marcos verbal instruction and memorandum, Tabuena, with the help of Gerardo G.
Dabao and Adolfo Peralta, caused the release of P55 Million of MIAA funds by
means of three (3) withdrawals. On 10 January 1986, the first withdrawal was made
for P25 Million, following a letter of even date signed by Tabuena and Dabao
requesting the PNB extension office at the MIAA the depository branch of MIAA
funds, to issue a managers check for said amount payable to Tabuena. The check was
encashed, however, at the PNB Villamor Branch. Dabao and the cashier of the PNB
Villamor branch counted the money after which, Tabuena took delivery thereof. The
P25 Million in cash was delivered on the same day to the office of Mrs. Gimenez.
Mrs. Gimenez did not issue any receipt for the money received. Similar circumstances
surrounded the second withdrawal/encashment and delivery of another P25 Million,
made on 16 January 1986. The third and last withdrawal was made on 31 January
1986 for P5 Million. Peralta was Tabuenas co-signatory to the letter- request for a
managers check for this amount. Peralta accompanied Tabuena to the PNB Villamor
branch as Tabuena requested him to do the counting of the P5 Million. After the
counting, the money was loaded in the trunk of Tabuenas car. Peralta did not go with
Tabuena to deliver the money to Mrs. Gimenez office. It was only upon delivery of
the P5 Million that Mrs. Gimenez issued a receipt for all the amounts she received
from Tabuena. The receipt was dated January 30,1986. Tabuena and Peralta were
charged for malversation of funds, while Dabao remained at large. One of the justices
of the Sandiganbayan actively took part in the questioning of a defense witness and of
the accused themselves; the volume of the questions asked were more the combined
questions of the counsels. On 12 October 1990, they were found guilty beyond
reasonable doubt. Tabuena and Peralta filed separate petitions for review, appealing
the Sandiganbayan decision dated 12 October 19990 and the Resolution of 20
December 1991.

Issue:
Whether or not petitioners are guilty of the crime of malversation.

Held:
Luis Tabuena and Adolfo Peralta are acquitted of the crime of malversation. Tabuena
acted in strict compliance with the MARCOS Memorandum. The order emanated
from the Office of the President and bears the signature of the President himself, the
highest official of the land. It carries with it the presumption that it was regularly
issued. And on its face, the memorandum is patently lawful for no law makes the
payment of an obligation illegal. This fact, coupled with the urgent tenor for its
execution constrains one to act swiftly without question. Records show that the
Sandiganbayan actively took part in the questioning of a defense witness and of the
accused themselves. The questions of the court were in the nature of cross
examinations characteristic of confrontation, probing and insinuation. Tabuena and
Peralta may not have raised the issue as an error, there is nevertheless no impediment
for the court to consider such matter as additional basis for a reversal since the settled
doctrine is that an appeal throws the whole case open to review, and it becomes the
duty of the appellate court to correct such errors as may be found in the judgment
appealed from whether they are made the subject of assignments of error or not.

Das könnte Ihnen auch gefallen